Categories
Economists Gender Harvard Radcliffe

Harvard/Radcliffe Economics Alumna, Rita Ricardo Campbell, 1946.

 

In the last post we met the 1948 Harvard economics Ph.D. alumnus, W. Glenn Campbell. Now it is time to meet his wife and fellow economist, Rita Ricardo (Radcliffe economics Ph.D., 1946). 

I have stumbled upon statements claiming that Rita was a direct descendent of David Ricardo, but they are incorrect. As she herself correctly wrote in the following letter to the White House (she was actively seeking to follow Martin Feldstein as Chairman of the Council of Economic Advisers under Ronald Reagan), she was a “collateral descendant” of that great classical economist, David Ricardo.

For exercise I climbed the Ricardo family tree to establish the degrees of separation between Rita and David, with whom she was indeed distantly related. She was clearly proud enough to flaunt her Ricardian pedigree professionally. My executive summary of the genealogical bottom line: Rita’s great-great grandfather was a third cousin of David Ricardo, and you can count the links yourself below. The main source used is the Lewis Family Tree Project at ancestry.com.

_______________

From a letter by Rita Ricardo-Campbell
to Michael K. Deaver,
Deputy Chief of Staff, White House.
November 21, 1984

“As a collateral descendant of the famous British economist David Ricardo, (that incidentally qualifies me as an Hispanic under the law!) I note that the well known Ricardian theory of the debt supports the President’s economic policy which I fully endorse.”

Source:  Ronald Reagan Presidential Library.

_______________

Radcliffe Ph.D. Thesis

Doctor of Philosophy
Degree awarded October 1946

Rita Ricardo Campbell, A.M.

Subject, Economics. Special Field, Labor Problems.
Dissertation, “Annual Wage and Employment Guarantee Plans”

Source:  Reports of Officers Issue, 1946-47 Sessions. Official Register of Radcliffe College Vol. XIII, No. 6 (December, 1947), p. 21.

_______________

Hoover Institution Obituary

Rita Ricardo-Campbell
1920-2016

The Hoover Institution announced today that renowned economist and senior fellow Rita Ricardo-Campbell died on March 7, 2016, at the age of ninety-five.

“Rita Ricardo-Campbell will be remembered for her meaningful contribution to health care and Social Security research.  While the loss is great, it is heartening that her legacy will live on through the Hoover Institution’s Glenn Campbell and Rita Ricardo Campbell National Fellows program,” stated Tom Gilligan, Director, Hoover Institution.

Ricardo-Campbell’s depth of experience extended to both the private and public sectors. She served as a director of the Gillette Company, the Watkins Johnson Company, and the Samaritan Medical Management Group. On the public side, she was a member of the President’s Economic Policy Advisory Board (1981-1989), a member of the National Endowment for the Humanities (1982-1988), a member of the President’s Committee on the National Medal of Science (1981 and 1991), and a member of the Advisory Council on Social Security (1974-1975).  She held teaching posts at Harvard and Tufts Universities before becoming an economist on the Wage Stabilization Board in Washington, DC, and subsequently as an economist for the House Ways and Means Committee.

“The impact of Rita’s work is well understood.  But what people don’t know is that Rita was a true pioneer, ahead of her time,” said Ed Lazear, Senior Fellow, Hoover Institution.  “She was the first female professor of economics at Harvard and throughout the years had significant influence on political leaders, all while raising her beautiful family.  She was an inspiration and will be missed by many.”

Ricardo-Campbell was a prominent writer, authoring a number of reputable books:  Social Security: Promise and Reality; The Economics and Politics of Health; Issues in Contemporary Retirement(coedited with Hoover Institution’s Edward Lazear); Aging: Social Security and Medicare; Below-Replacement Fertility in Industrial Societies; and Women and Comparable Worth.

A native of Boston, Massachusetts, Ricardo-Campbell received her bachelor’s of science degree from Simmons College and master’s and PhD degrees from Harvard University. She was preceded in death by her husband, former Hoover Institution Director W. Glenn Campbell. She is survived by three daughters, Diane Rita Campbell, Barbara Lee Gray and Nancy Elizabeth Yaeger, and four grandchildren.

Ricardo-Campbell’s research papers are available at the Hoover Institution Archives.

Source: https://www.hoover.org/press-releases/hoover-institution-celebrates-life-fellow-rita-ricardo-campbell

Image Source:  Rita Ricardo, Class of 1941. Simmons College Yearbook Microcosm, p. 64.

_______________

The 9-generation line from Samuel ‘Moses’ Israel Ricardo
to Rita Ricardo-Campbell

Samuel ‘Moses’ Israel Ricardo (est1624-ca1692) and Diana Israel (1628-1709)

Daniel ‘Samuel’ Israel Ricardo (1657-) and Rebecca ‘Jacob’ Nunes Mendes (1660-1722)

Benjamin ‘Daniel’ Israel Ricardo (1694-1768) and Gracia ‘Isaac’ Saraga (1701-)

Daniel ‘Benjamin’ Israel Ricardo (1722-1787) and Rachel ‘Salomon’ de Rocamora (1745-1787)

Abraham Daniel Ricardo, 1786-1842 and Benvenida ‘Abraham, David’ Senior Coronel (1789-1828)

Daniel Abraham Ricardo 1812-1871 (Birth in Amsterdam) and Jetje Catarina ‘Elias’ Barentz (1811-)

Aaron Daniel Ricardo (1852-1920?) born in Amsterdam, died in London and Rebecca ‘Abraham’ Lopes Salzedo (1850-1900)

David ‘Aaron’ Ricardo (1878-) and Elizabeth Jones (1900-), both born in England

Rita Ricardo-Campbell (16 Mar 1920 (Boston)-July 3, 2016 (Stanford)

 

The 5 generation line from Samuel ‘Moses’ Israel Ricardo
to David Ricardo

Samuel ‘Moses’ Israel Ricardo (est1624-ca1692) and Diana Israel (1628-1709)

David ‘Samuel’ Israel Ricardo (1652-) and Estrella (Strellia) ‘Joseph’ Amadeos (1663-)

Joseph ‘David’ Israel Ricardo (1699-1762) and Hanna ‘Abraham’ Abas (1705-1781)

Abraham ‘Joseph’ Israel Ricardo (1735-1812) and Abigail ‘Abraham’ del Valle (1753-)

David ‘Abraham’ Ricardo (1722-1823) and Priscilla Wilkinson(1775-)

Categories
Economists Harvard Stanford

Harvard. Economics Ph.D. Alumnus, W. Glenn Campbell, 1948

 

In the last post we came across a young co-instructor for Harvard’s Principles of Economics course during the 1949-50 academic year, Wesley Glenn Campbell. This 1948 Harvard Ph.D. went on to become “the man who built the Hoover Institution“.

Campbell was born April 19, 1924 in Lobo Township in Ontario, Canada and died November 24, 2001 in Los Altos Hills, California. He was married to fellow economist, Rita Ricardo-Campbell.

____________________

Ph.D. Thesis

Wesley Glenn Campbell, B.A. (Univ. of Western Ontario) 1944, A.M. (Harvard Univ.) 1946.

Subject, Economics. Special Field, Public Finance. Thesis, “Impact of Social Security Expenditures on Canadian Government Finance.”

Source: Harvard University. Report of the President of Harvard College, 1947-48, p. 174.

____________________

From the national press

“…his conservative thinking prompted Harvard colleagues, in his view, to force him out for political reasons. Dr. Campbell served as research economist for the U.S. Chamber of Commerce from 1951 to 1954, then as director of research for the American Enterprise Association in Washington, until Hoover tapped him to move to Stanford.”

Source:  From the Washington Post obituary (December 1, 2001).

“President Herbert Hoover appointed Dr. Campbell to head the Hoover Institution in an effort to keep what Hoover called left-wingers from gaining control of it. Three months after Mr. Campbell’s appointment, the Stanford faculty tried to end the institution’s autonomy and put it under direct university control. Hoover, then 86, ended this initiative with the flick of his pen.”

Source: From the New York Times obituary (November 28, 2001)

____________________

Stanford Obituary

W. Glenn Campbell, the outspoken director of the Hoover Institution who built it into an internationally known think tank, died Nov. 24 of a heart attack at his home in Los Altos Hills. He was 77.

A funeral service will be held at 1 p.m. Thursday at the Los Altos Chapel of Spangler Mortuaries at 399 San Antonio Road. Plans for a memorial service on campus are pending.

Hoover Director John Raisian said that his predecessor, who retired in 1989, “served this institution magnificently. He was an institution builder, an advocate of freedom and a contributor to our nation’s well-being.”

In 1960, Campbell, a free-market economist, was handpicked by former President Herbert Hoover to run his library. Under Campbell’s 29-year leadership, the Hoover Institution on War, Revolution and Peace grew into a powerful think tank. Its endowment grew from $2 million to more than $125 million and it more than tripled in size physically.

“He was the man who built the Hoover Institution,” said Senior Fellow Melvyn Krauss. “And he was an early founder of think tanks in the United States. He was a terrific fundraiser and he brought outstanding people to Hoover.”

Former U.S. Secretary of State George Shultz said Campbell’s guiding idea in both politics and economics was his “continual fight for freedom. That led him to all sorts of positions that were controversial at the time, but not anymore,” Shultz said. “He was a dedicated anti-Communist and a severe critic of the Soviet Union. Now people say, ‘You were right after all.'” He was also dedicated to market solutions, not government solutions, to economic problems, Shultz said.

Senior Fellow Bruce Bueno de Mesquita said Campbell successfully turned an obscure library into one of the world’s leading think tanks. “We had possibly one of the largest sets of Nobel laureates in economics affiliated with Hoover,” he said. Campbell chafed at the description of Hoover as a conservative think tank, Bueno de Mesquita said: “Glenn was much broader in his vision. He hired extraordinary people. Glenn did have a political side but also an academic side.” The fellows included economist Milton Friedman; physicist Edward Teller, designer of the hydrogen bomb; Soviet expert Robert Conquest; and Shultz.

Krauss said Campbell successfully hired high-profile stars, such as Friedman, after they retired from other institutions. At first, “it was tough for us to get mainline people,” he said. “Glenn was ingenious in his strategy of creating ‘over-age’ appointments.” And by establishing joint appointments between Hoover and Stanford departments, a move that allowed scholars to earn a higher salary, Krauss said, the university became more competitive in attracting top people.

Campbell was a longtime supporter of former President Ronald Reagan, whom he met when the one-time actor ran for governor of California. In 1968, Reagan appointed Campbell to the Board of Regents of the University of California. He served as a regent for 28 years, often clashing with UC’s administration. In 1969, for example, he sided with Reagan in his crackdown on student protests over the Vietnam War.

Krauss said that Campbell’s close relationship with Reagan benefited Hoover. “When he became president, we had a bonanza,” he said. Many of the fellows went on to serve in Washington, D.C., and helped create the ideological framework for the “Reagan revolution.”

Referring to the Hoover Institution book, The United States in the 1980s, former Soviet leader Mikhail Gorbachev said, “We have read this book and have watched all its programs become adopted by the Reagan administration.”

Campbell’s close relationship with the Republican Party, however, often caused him to have run-ins with Stanford. In 1987, the university thwarted Campbell’s effort to bring the Ronald Reagan Presidential Library and Public Affairs Center to campus. Stanford’s trustees extended to Reagan an invitation to build the library, but not the public affairs center, concerned that the latter would turn into a conservative think tank. Reagan initially accepted the offer but later established his library in Southern California.

A year later, the trustees, citing a mandatory retirement age policy, informed Campbell that he would have to retire in 1989, the year he turned 65. Campbell fought to stay on but, after securing a generous retirement package, stepped down and was appointed counselor to the director. In 1994, Campbell was named director emeritus.

Campbell was born on a farm in Lobo Township in Ontario, Canada. He graduated from the University of Western Ontario in 1944 with honors in economics and political science. In 1948, he graduated from Harvard University with a doctorate in economics.

Campbell is survived by his wife of 55 years, Rita Ricardo-Campbell, a Hoover senior fellow emerita; sisters Marjorie Wyatt and Evelyn McClary of Ontario, Canada; daughters Nancy Yaeger of Los Angeles, Diane Campbell of Irvine and Barbara Gray of Walnut Creek; and four grandchildren.

Source: Lisa Trei, “Glenn Campbell, former Hoover director, dead at 77”, Stanford Report, November 28, 2001.

Image source: Image reduced from portrait ca. 1965 of Wesley Glenn Campbell in the Campbell Family Tree 2 at ancestry.com

Categories
Harvard Suggested Reading Syllabus Undergraduate

Harvard. Principles of Economics. Course outline, readings, exam questions, 1949-50

 

Of particular interest in this two-track (for economics concentrators and non-concentrators, respectively) principles of economics course is that the Keynesian Cross chapter (XII) of Paul Samuelson’s new textbook Economics was assigned in the concentrators’ version.

The course was taught by Professor Burbank and the newly minted Harvard Ph.D. Wesley Glenn Campbell who would later be hand-picked by former President Herbert Hoover to head to the Hoover Institution.

________________________

Course Description

ECONOMICS
1949-50

Primarily for Undergraduates

Economics 1 (formerly Economics Aa and Ab). Principles of Economics

Full course. Tu., Th., Sat., at 11. This course is conducted by sections. It will be divided into sections for concentrators and for non-concentrators. There will be sections at other hours. (Radcliffe sections will meet Tu., Th., Sat., at 11 and at such other times as the enrolment may justify.) Professor BURBANK, Dr. [Wesley Glenn] CAMPBELL [Harvard Ph.D., 1948], and other MEMBERS OF THE DEPARTMENT.

Economics 1 may be taken by properly qualified Freshmen with the consent of the instructor.

Economics 1 provides an introduction to the principles required for the analysis of economic problems. The development of principles in the main fields of economics and the study of economic organization give the non-concentrator a background for the understanding of economic problems and are indispensable for the concentrator’s further work in advanced courses.

 

Source:  Harvard University Archives. Syllabi, course outlines and reading lists in Economics, 1895-2003. Box 4, Folder “Economics, 1949-1950 (1 of 3)”.

________________________

Course Enrollment

[Economics] 1 (formerly Economics Aa and Ab). Principles of Economics. (Full Co.) Professor Burbank, Dr. Campbell, and other Members of the Department.

(Fall) Total 441: 1 Graduate, 16 Seniors, 68 Juniors, 220 Sophomores, 110 Freshmen, 21 Radcliffe, 5 Special.
(Spring) Total 434: 1 Graduate, 18 Seniors, 72 Juniors, 240 Sophomores, 73 Freshmen, 26 Radcliffe, 4 Special.

 

Source: Harvard University. Annual Report of the President of Harvard College, 1949-50, p. 72.

 

________________________

ECONOMICS I—CONCENTRATORS
1949-50
First Half

Sources:

Benham and Lutz Economics, American Edition (1941)
*Bowman and Bach Economic Analysis and Public Policy, Second Edition (1949)
Burns, Neal & Watson Modern Economics(1948)
Hart, A.G. Money, Debt, and Economic Activity(1948)
Merrill, Lynch, et al, How to Read a Financial Report
*Peach and Krause Basic Data of the American Economy, Revised Edition, (1949)
Peterson, S. Economics(1949)
Schumpeter, J. A. Capitalism, Socialism, and Democracy
Slichter, S. H. Modern Economic Society(1931)
Slichter, S. H. The American Economy(1948)
Williamson, H. F. The Growth of the American Economy

*To be purchased by students.

 

PART I. Introduction

  1. The Economic Problem
    Benham: Ch. 1, General Survey
  2. Economic Institutions and Economic Development
    Burns: Ch. 2, Change and Growth in the Economy
    Bowman & Bach: Ch. 6, Economic Analysis and Public Policy

PART II. National Income, Money, Banking and Price Levels

  1. National Income
    Burns: Ch. 4, National Income and National Output
    Peach & Krause: Section I, National Income
  2. Money, Banking and Price Levels
    Merrill, Lynch, et al.: How to Read A Financial Report
    Peach & Krause: Section 4, Money and Banking
    Peterson: Ch. 10, Exchange Media. Hand-to-Hand Money
    Bowman & Bach: Ch. 10, The Banking System, the Money Supply, and Investment; Ch. 11, The Government and the Money Supply
    R.B.: Banking and Monetary Statistics, Section 10, pp. 360-366
    National Debt Series: 2, Our National Debt and the Banks; 3, Our National Debt and Interest Rates; 6, Our National Debt and Life Insurance
    Hart: Ch. 10, Inflation and Deflation

PART III. Role of Markets in the Allocation of Resources and the Determination of Relative Prices

  1. Markets—An Introduction to the Problems of Production, Distribution, Exchange and Consumption
    Bowman & Bach: Ch. 2, Income and Consumption; Ch. 3, The Economic System—A Summary View; Ch. 4, Private Enterprise, Profits, and the Price System; Ch. 5, Business Enterprise in the Modern Economy—omit appendix
  2. Price Determination and Resource Allocation
    Bowman & Bach: Book III, Production, Individual Prices, and the Allocation of Resources
    Williamson: Ch. 25, The Location of Economic Activity
    Benham: Ch. 2, Markets, pp. 38-46
    Slichter: Ch. 10, Speculative Production, pp. 215-221
  3. Public Control of Markets
    Bowman & Bach: Ch. 33, Government Policy and Business Practice
    Schumpeter: Ch. 8, Monopolistic Practices
    Peterson: Ch. 23, Market Control Policies in the United States, pp. 618-631
    Peach & Krause: Section 9, Agriculture
  4. The Productive Performance of the American Economy
    Slichter: Ch. 1, The American Economy; Ch. 6, How Good is the American Economy
    Peach & Krause: Section 2, Population and the Working Force in the United States
    Peach & Krause: Section 3, National Resources

 

Source:  Harvard University Archives. Department of Economics. Course Reading lists, syllabi, and exams 1913-1992, Box 2, Folder “Lecture Schedules and Reading Lists, 1942-1970”, Subfolder “49-55”.

________________________

ECONOMICS I—NON-CONCENTRATORS
1949-50
First Half

Sources:

Arnold, T. The Bottlenecks of Business(1940)
Benham and Lutz Economics, American Edition (1941)
Bowman and Bach Economic Analysis and Public Policy, Second Edition (1949)
*Federal Reserve System Federal Reserve Charts on Bank Credit, Money Rates and Business(Latest edition)
Hart, A.G. Money, Debt, and Economic Activity(1948)
Johnson, E. A. J. Some Origins of the Modern Economic World
Merrill, Lynch, et al., How to Read a Financial Report
*Peterson, S. Economics(1949)
Slichter, S. H. Modern Economic Society(1931)
*Slichter, S. H. The American Economy(1948)
Williamson, H. F. The Growth of the American Economy
*Wright, D. M. Democracy and Progress

*To be purchased by students.

 

PART I. Introduction

  1. The Economic Problem
    Benham: Ch. 1, General Survey
  2. Economic Institutions and Economic Development—An Historical Approach
    Johnson: Ch. 2, The Late-Medieval Background; Ch. 3, The Emergence of Capitalism; Ch. 4, The Beginnings of Scientific Technology
    Williamson: Ch. 3, The Organization of Production During the Colonial Period
    Bowman & Bach: Ch. 6, Economic Analysis and Public Policy

PART II. The Role of Markets in the Allocation of Resources and the Determination of Relative Prices

  1. A Comprehensive View of the Market System
    Peterson: Ch. 2, The Occupational and Industrial Structure; Ch. 3, Production and Income—Individual and National; Ch. 4, Framework and Problems of the Economic System
  2. The Determinants of Productive Power and the Organization of Production Under Capitalism
    Peterson: Ch. 5, Natural and Human Resources; Ch. 6, Capitalistic Production; Ch. 7, The Organization of Production; Ch. 8, Business Enterprise and the Corporate Form
    Merrill, Lynch, et al.: How to Read a Financial Report
    Peterson: Ch. 9, Finance, pp. 207-214 and 221-236
    Williamson: Ch. 14, The Capital Markets, 1789-1860; Ch. 28, The Investment Market After the War Between the States
  3. Price Determination and Resource Allocation
    Peterson: Ch. 17, The Role of Prices; Ch. 18, Supply, Demand, and Market Price
    Benham: Ch. 2, Markets, pp. 38-46
    Slichter: Ch. 10, Speculative Production, pp. 215-221
    Peterson: Ch. 19, Nature and Role of Demand and its Elasticity; Ch. 20, Cost and the Expansion and Contraction of Industries
    Williamson: Ch. 25, The Location of Economic Activity
    Peterson: Ch. 21, Output from Existing Capacity
  4. Public Regulation of Markets
    Peterson: Ch. 22, Monopoly and the Public Interest
    Williamson: Ch. 30, Industrial Concentration and Government anti-Trust Policy
    Arnold: Ch. 2, How Restraints of Trade Affect Your Standard of Living; Ch. 3, How Restraints of Trade Unbalance the National Budget; Ch. 7, Procedure under the Sherman Act; Ch. 8, The Clarification of Law; Appendix I
    Peterson: Ch. 23, Market Control Policies in the United States
    Wright: Ch. 8, The Problems of Competition
  5. The Production and Distribution of Wealth
    Slichter: Ch. 1, The American Economy; Ch. 6, How Good is the American Economy
    Wright: Ch. 7, Economic Goals and the Distribution of Wealth

PART III. Money, Banking, Price Levels and the National Income

  1. Money, Banking and Price Levels
    Peterson: Ch. 10, Exchange Media. Hand-to-Hand Money
    Bowman & Bach: Ch. 10, The Banking System, the Money Supply, and Investment; Ch. 11, The Government and the Money Supply
    R.B.: Banking and Monetary Statistics, Section 10, pp. 360-366
    National Debt Series: 2, Our National Debt and the Banks; 3, Our National Debt and Interest Rates; 6, Our National Debt and Life Insurance
    Hart: Ch. 10, Inflation and Deflation
  2. Mechanics of the International Monetary Exchange
    Benham: Ch. 26, Balance of Payments
    Hart: Ch. 15, The Foreign Exchange Market
    Benham: Ch. 27, Free Exchange Rate; Ch. 28, The Gold Standard

 

Source:  Harvard University Archives. Department of Economics. Course Reading lists, syllabi, and exams 1913-1992, Box 2, Folder “Lecture Schedules and Reading Lists, 1942-1970”, Subfolder “49-55”.

________________________

ECONOMICS I—CONCENTRATORS
1949-50
Second Half

Sources:

Benham and Lutz Economics, American Edition (1941)
*Bowman and Bach Economic Analysis and Public Policy,Second Edition (1949)
**Committee for Economic Development The Uses and Dangers of Direct Controls in Peacetime
**Economic Outlook Consumers, Workers Pay Cost of New Factories
Hart, A. G. Money, Debt, and Economic Activity(1948)
**International Bank for Reconstruction and Development Fourth Annual Report, 1948-49
**International Monetary Fund Annual Report, 1949
**Murray, P. The Steelworkers’ Case for Wages, Pensions and Social Insurance
*Peach and Krause Basic Data of the American EconomyRevised Edition (1949)
Peterson, S. Economics(1949)
Samuelson, P. Economics
Slichter, S. H. Basic Criteria Used in Wage Negotiations
**Slichter, S. H. Profits in a Laboristic Society
**Slichter, S. H. The Taft-Hartly Act
**Steel Industry Board Report to the President of the United States
**Voorhees, E. M. Statement before the Presidential Steel Board
Wright, D. M. Democracy and Progress

* To be purchased by students
**To be handed out in section meeting.

 

PART IV. The Distribution of Income

  1. Introduction
    Bowman & Bach: Ch. 28, Introduction to the Study of Income Distribution
  2. Personal Income Distribution
    Bowman & Bach: Ch. 29, Personal Income Distribution in the United States
    Wright: Ch. 7, Economic Goals and the Distribution of Wealth
  3. Determination of Returns to the Factors of Production
    Bowman & Bach: Ch. 30, Wage and Salary Income; Ch. 32, Property Income
  4. Labor Organization and Labor Markets
    Bowman & Bach: Ch. 31, The Economics of Labor Unionism
    Slichter: Basic Criteria Used in Wage Negotiations, pp. 7-31, and 36-40
    Bowman & Bach: Ch. 35, Government Policy and Labor, pp. 651-673
    Slichter, The Taft-Hartley Act
  5. The Wages, Pensions, Prices and Profits Controversy
    Economic Outlook: Consumers, Workers Pay Cost of New Factories
    Slichter: Profits in a Laboristic Society
    Murray, The Steelworkers’ Case for Wages, Pensions and Social Insurance, pp. 9-29
    Voorhees, Statement before the Presidential Steel Board
    Steel Industry Board, Report to the President of the United States, pp. 1-11

PART V. International Economic Problems

Benham: Ch. 25, The Theory of International Trade; Ch. 26, Balances of Payments
Peach & Krause: Section 5, International Trade and Finance
Hart: Ch. 15, The Foreign Exchange Market
Benham: Ch. 27, Free Exchange Rates; Ch. 28, The Gold Standard; Ch. 29, Exchange Control; Ch. 30, Import Duties and Quotas
Hart: Ch. 18, International Monetary Cooperation
International Monetary Fund: Annual Report, 1949, pp. 1-46
International Bank for Reconstruction and Development: Fourth Annual Report, 1948-49, pp. 7-37

PART VI. Public Finance and the Economic Problem

Peach & Krause: Section 6, Government Expenditures, Tax Collections, Public and Private Debt
Bowman & Bach: Ch. 36, Introduction to the Public Economy; Ch. 37, Public Expenditures; Ch. 38, Public Revenues—Taxation; Ch. 39, Taxation (Continued)
Peterson: Ch. 30, Public Policy and the Distribution of Income

PART VII. The Nature of Economic Fluctuations and Policies Directed Toward Their Control

Samuelson: Ch. 12, Saving and Investment
Peach & Krause, Review Section 1, National Income
Hart: Review Ch. 10, Inflation and Deflation
Bowman & Bach: Ch. 13, The Rate of Economic Growth; Ch. 14, Economic Fluctuations
Peach & Krause: Section 7, Price Levels and Business Fluctuations
Wright: Ch. 6, Progress and Instability
Bowman & Bach: Ch. 40, Monetary Policy and Economic Stabilization; Ch. 41, Fiscal Policy and Economic Stabilization; Ch. 42, Antimonopoly Measures, Wage-Price Policy, and Direct Controls
C.E.D.: The Uses and Dangers of Direct Controls in Peacetime

 

Source:  Harvard University Archives. Department of Economics. Course Reading lists, syllabi, and exams 1913-1992, Box 2, Folder “Lecture Schedules and Reading Lists, 1942-1970”, Subfolder “49-55”.

________________________

ECONOMICS I—NON-CONCENTRATORS
1949-50
Second Half

Sources:

Benham and Lutz Economics, American Edition (1941)
Bowman and Bach Economic Analysis and Public Policy,Second Edition (1949)
**Economic Outlook Consumers, Workers Pay Cost of New Factories
Hart, A. G. Money, Debt, and Economic Activity(1948)
**International Bank for Reconstruction and Development Fourth Annual Report, 1948-49
**International Monetary Fund Annual Report, 1949
Jewkes, J. Ordeal by Planning(1948)
*Peterson, S. Economics(1949)
*Schumpeter, J. A. Capitalism, Socialism and Democracy(1947)
*Slichter, S. H. The American Economy(1948)
Slichter, S. H. Basic Criteria Used in Wage Negotiations
**Slichter, S. H. Profits in a Laboristic Society
Sweezy, P. M. Socialism
*Wright, D. M. Democracy and Progress

* To be purchased by students
**To be handed out in section meeting.

 

PART IV. The Distribution of Income

  1. Personal Income Distribution
    Peterson: Ch. 24, Inequality—Extent and Significance; Ch. 25, Inequality in the Return from Labor
  2. Determination of Returns to the Factors of Production
    Peterson: Ch. 26, Productivity and Income; Ch. 28, The Basis of Property Incomes; Ch. 29, Profits, Interest, and Wealth
  3. Labor Organization and Labor Markets
    Bowman & Bach: Ch. 31, The Economics of Labor Unionism, pp. 492-501
    Peterson: Ch. 27, Wage-raising Policies and Practices
    Slichter: Basic Criteria Used in Wage Negotiations, pp. 7-31, and 36-40
    Bowman & Bach: Ch. 35, Government Policy and Labor, pp. 651-681
    Slichter: Ch. 2, Co-operation or Conflict in American Industry
  4. The Wages, Prices and Profits Controversy
    Economic Outlook: Consumers, Workers Pay Cost of New Factories
    Slichter: Profits in a Laboristic Society

PART V. International Economic Problems

Benham: Ch. 25, The Theory of International Trade; Review Chs. 26, 27, 28
Hart: Review, Ch. 15
Benham: Ch. 29, Exchange Control; Ch. 30, Import Duties and Quotas
Hart: Ch. 18, International Monetary Cooperation
International Monetary Fund: Annual Report, 1949, pp. 1-46
International Bank for Reconstruction and Development: Fourth Annual Report, 1948-49, pp. 7-37

PART VI. Public Finance and the Economic Problem

Bowman & Bach: Ch. 36, Introduction to the Public Economy; Ch. 37, Public Expenditures; Ch. 38, Public Revenues—Taxation; Ch. 39, Taxation (Continued)
Peterson: Ch. 30, Public Policy and the Distribution of Income

PART VII. The Nature of Economic Fluctuations and Policies Directed Toward Their Control

Peterson: Ch. 14, Total Demand and the Depression Problem; Ch. 15, Cyclical Fluctuations
Wright: Ch. 6, Progress and Instability
Slichter: Ch. 3, The Problem of Economic Stability
Wright: Ch. 11, Three Plans

PART VII. The Prospects for Economic Progress under Capitalism and Other Systems

Schumpeter: Part II, Can Capitalism Survive
Wright: Ch. 1, Science, Democracy, and Capitalism; Ch.2, The Moral Dilemma of Progress; Ch. 3, The Meaning and the Method of Democratic Progress; Ch.4, Political Democracy and the Alternatives to Competition
Schumpeter: Part III, Can Socialism Work?
Sweezy: Ch. 10, Can Socialism Provide Incentives to Work and to Efficiency?; Ch. 12, Are Socialism and Freedom Compatible?
Jewkes: Ch. 1, The Spread of Fashion; Ch.2, Is the Business Man Obsolete; Ch. 5, Confusion Among the Planners; Ch. 6, Planners as a Species; Ch. 7, Planning as a Scientific Method; Ch. 8, Planning and Prosperity; Ch. 9, Planning and Economic Stability; Ch. 10, Planning and Freedom

 

Source:  Harvard University Archives. Department of Economics. Course Reading lists, syllabi, and exams 1913-1992, Box 2, Folder “Lecture Schedules and Reading Lists, 1942-1970”, Subfolder “49-55”.

________________________

1949-50
HARVARD UNIVERSITY
ECONOMICS I
Non-Concentrators

Mid-Year Examination
January, 1950

I
(One hour and a half)
Answer both questions

  1. A member of the Board of Governors of the Federal Reserve System has recently advised Congress that the policy of the Treasury has made it impossible for the Federal Reserve authorities to use their powers as controllers of the country’s money supply. Explain carefully why Treasury and Federal Reserve policies must be coordinated and in what ways they are likely to come in to conflict. Illustrate by reference to the national debt and other problems which arose in the war and the postwar periods.
  2. The problem of the allocation of scarce resources among a multitude of possible uses is one which is largely solved automatically in our economy.
    Explain how this problem is solved. Give careful attention to the role of and inter-relationships among each of the following: consumer decisions, producer decisions and markets.

II
(One hour and a half)
Answer any THREE questions

  1. The monetary control authorities generally attempt to control the level of prices and the level of income through control of the supply of money. Using the equation of exchange as an analytic framework, analyze how a policy which changes the supply of money might work out.
  2. Answer either (a) or (b) of the following
    1. Distinguish “rate level” from “rate structure.” Discuss the criteria relied on by regulatory commissions in determining each for a public utility, noting the major problems involved.
    2. What are the major economic arguments for and against monopoly? In the light of these arguments what elements do you think should be contained in any balanced government policy toward monopoly?
  3.      aExplain the relationship between gross and net national product; between national income and aggregate personal income.
    1. Discuss a purpose for which each one of the above aggregates can be used.
    2. In the light of the above explanation and additional pertinent facts comment on the following statement: “A comparison of national income at the depth of a depression with that during a period of prosperity overstates the impact of the depression on the consuming public.”
  4. Answer TWO of the following:
    1. Explain how speculative markets control the rate of use of periodically produced goods.
    2. Restate the Malthusian thesis (law of population) using the principle of diminishing returns.
    3. Distinguish the short-run stabilization and long-run adjustment of the market for farm products. Consider both the objectives and the implied policies.
    4. Discuss the respective roles of technological change and savings and capital accumulation in the emergence of modern economic society.

Source:  Harvard University Archives. Department of Economics. Course Reading lists, syllabi, and exams 1913-1992, Box 2, Folder “Economics 1, Exams 1939-1962”.

________________________

1949-50
HARVARD UNIVERSITY
ECONOMICS I
Concentrators

Mid-Year Examination
January, 1950

 

I
(One hour and a half)
Answer both questions

  1. A member of the Board of Governors of the Federal Reserve System has recently advised Congress that the policy of the Treasury has made it impossible for the Federal Reserve authorities to use their powers as controllers of the country’s money supply. Explain carefully why Treasury and Federal Reserve policies must be coordinated and in what ways they are likely to come in to conflict. Illustrate by reference to the national debt and other problems which arose in the war and the postwar periods.
  2. Consumers’ preferences change, thus increasing the demand for a certain product which is produced under conditions of pure competition. Trace in precise fashion the results of this increase in demand on the output of the individual firms and of the industry, and on the price of the product:
    1. in the short run,
    2. in the long run.

 

II
(One hour and a half)
Answer any THREEquestions

  1. The monetary control authorities generally attempt to control the level of prices and the level of income through control of the supply of money. Using the equation of exchange as an analytic framework, analyze how a policy which changes the supply of money might work out.
  2. What are the major economic arguments for and against monopoly? In the light of these arguments what elements do you think should be contained in any balanced government policy toward monopoly?
  3. Answer TWO of the following:
    1. Discuss three important factors determining the location of economic activity.
    2. “When there is oligopoly, even without collusive agreements, price competition will tend to be ‘nonaggressive’, and price will usually be higher than otherwise.” Discuss.
    3. “Competition on a nonprice basis has become more and more important in recent years.” Discuss the effects of this trend on the allocation of resources.
    4. Discuss the process of hedging in a commodity market and its significance to the non-speculative businessman.
  4. Define Gross National Output (Product), National Income, and Income Payments (Personal Income).
    1. What is the general use of these concepts and how might each one be used specifically?
    2. How is Gross national Output related to Aggregate Demand or Expenditure?
    3. How will the relation between National Income and income Payments vary in prosperity and depression?
    4. Can we place great reliance on these concepts as measures of economic welfare?

Source:  Harvard University Archives. Department of Economics. Course Reading lists, syllabi, and exams 1913-1992, Box 2, Folder “Economics 1, Exams 1939-1962”.

________________________

1949-50
HARVARD UNIVERSITY
ECONOMICS I
Non-Concentrators

Final Examination
June, 1950

I
(One hour and a half)
Answer both questions

  1. Investment is often said to play a “strategic role” in the business cycle. What is meant by this statement? What are its implications for counter-cyclical policy?
  2. “Remuneration for labor services and a share in the social dividend are the only sources of personal income under socialism. Therefore, the socialist planners can ignore rent, interest, and profits even though they are fundamental to the functioning of a capitalist system.” Discuss.

II
(One hour and a half)
Answer both questions

  1. Without stating general conclusions as to the merits of either side, explain the basic issues involved in the dispute between labor and industry over wages, prices and profits.
  2. Discuss the elements to be considered in the establishment of a model tax system for the United States at the present level of expenditures. (This includes all levels of Government.)

III
(Thirty minutes)
Answer one question

  1. An adverse balance of payments can be corrected by (1) changes in exchange rates, (2) changes in prices and incomes, or (3) exchange and import controls.
    1. Discuss briefly how each of the above three methods may be used to correct a country’s adverse balance of payments.
    2. Discuss the extent to which the member countries of the International Monetary Fund may make use of the above three methods.
  2. Comment on the following statement: “The object of American tariff policy should be to impose sufficient duty on goods of every kind to equalize the cost of production at home and abroad.”

Source:  Harvard University Archives. Department of Economics. Course Reading lists, syllabi, and exams 1913-1992, Box 2, Folder “Economics 1, Exams 1939-1962”.

________________________

1949-50
HARVARD UNIVERSITY
ECONOMICS I
Concentrators

Final Examination
June, 1950

I
(One hour and a half)
Answer both questions

  1. Investment is often said to play a “strategic role” in the business cycle. What is meant by this statement? What are its implications for counter-cyclical policy?
  2. The establishment of product prices and of returns to factors of production are two sides of the same economic process.
    1. Analyze the forces of supply and demand which determine the return to a factor of production.
    2. Explain (in terms of producer and consumer decisions) how these returns determine and are determined by the prices of products.

II
(One hour and a half)
Answer both questions

  1. Without stating general conclusions as to the merits of either side, explain the basic issues involved in the dispute between labor and industry over wages, prices and profits.
  2. Discuss the elements to be considered in the establishment of a model tax system for the United States at the present level of expenditures. (This includes all levels of Government.)

III
(Thirty minutes)
Answer one question

  1. An adverse balance of payments can be corrected by (1) changes in exchange rates, (2) changes in prices and incomes, or (3) exchange and import controls.
    1. Discuss briefly how each of the above three methods may be used to correct a country’s adverse balance of payments.
    2. Discuss the extent to which the member countries of the International Monetary Fund may make use of the above three methods.
  2. Comment on the following statement: “The object of American tariff policy should be to impose sufficient duty on goods of every kind to equalize the cost of production at home and abroad.”

Source:  Harvard University Archives. Department of Economics. Course Reading lists, syllabi, and exams 1913-1992, Box 2, Folder “Economics 1, Exams 1939-1962”.

Image Source:  H. H. Burbank in the Harvard Class Album 1947.

Categories
Columbia Gender

Columbia. Faculty of Political Science Not Yet Supporting Admission of Women, 1892

 

It is not clear whether the undersigned were actually against the admission of women to the graduate courses offered by the Faculty of Political Science of Columbia University in 1892 or procedural sticklers navigating troubled waters (or both). In any event, this is a pretty curious document.

___________________

Names, Ranks, and Fields of Signers

Edmund Munroe Smith, Professor of Roman Law and Comparative Jurisprudence
Frank J. Goodnow, Professor of Administrative Law
Edwin R. A. Seligman, Professor of Political Economy and Finance and Secretary of the Faculty
William A. Dunning, Adjunct Professor of History
John Bassett Moore, Professor of History and Political Philosophy
Herbert L. Osgood, Adjunct Professor of History

___________________

Admission Interruptus

Columbia College
In the City of New York
School of
Political Science

Jan. 15, 1892

Seth Low, LL.D.,
President of Columbia College.

Dear Sir:

We, the undersigned members of the Faculty of Political Science, desire to withdraw for the present our assent, given separately and without consultation, to the admission of women to our University courses. It is evident to us, on reflection, that the admission of women to certain courses makes it very difficult to exclude them from any, and that the assent of each professor in so far prejudices the decision of all: and we think that a change of policy of such importance should be made only by the Faculty, and after general and full discussion.

We see moreover the possibility of great detriment to the work of the School of Political Science if this question should be determined without a degree of harmony in the Faculty which does not as yet exist.

Yours respectfully,
[signed]
Munroe Smith
Frank J. Goodnow
Edwin R. A. Seligman
Wm. A. Dunning
J. B. Moore
Herbert L. Osgood

Source: Columbia University, Rare Book and Manuscript Library. Columbia University Archives. Central Files 1890-. Box 339. Folder: “1.1.19; Smith, Munroe; 5/1891-11/1909”.

 

 

Categories
Bryn Mawr Gender Harvard Radcliffe

Harvard/Radcliffe. Economics PhD alumna, Ruth Jackson Woodruff, 1931

 

Besides the curricula of graduate education in economics, every so often Economics in the Rear-view Mirror presents the life-stories of men and women who have received a Ph.D. in economics. Where did they come from and where did they end up, along with all the stations in between. Today we meet Ruth Jackson Woodruff, a Radcliffe Ph.D. (1931). This was back in the day when Harvard and Radcliffe still differentiated their doctorates.

___________________

Doctor of Philosophy

Ruth Jackson Woodruff, A.M.

Subject, Economics. Special Field, Economic History since 1750. Dissertation “A History of the Hosiery Industry in the United States before 1890.”

Source:  Annual Reports of Radcliffe College for 1930-31 (February, 1932), p. 21.

___________________

Publications

Woodruff, R. (1921). A Classification of the Causes of Crime. Journal of the American Institute of Criminal Law and Criminology, 12(1), 105-109. [Written while still a student at Bryn Mawr College.]

Ruth Woodruff, “The Hosiery Industry,” Bulletin Series No. 5, Junior Employment Service, Board of Education of Philadelphia (Philadelphia, 1925).

Alexander, N., & Woodruff, R. (1940). Determinants of College Success. The Journal of Higher Education, 11(9), 479-485.

___________________

Life and career dates

December 21, 1898. Born in Scranton, Pennsylvania.

1919. Bryn Mawr, A.B.

1920. Bryn Mawr, A.M.

1927-28. Attended University of Pennsylvania.

1931. Radcliffe, Ph.D. in economics.

1932-1953. Dean of Women at the University of New Hampshire. [Began as assistant professor of economics in the College of Liberal Arts]

1954-1962. Professor of Economics in the College of Liberal of Arts of the University of New Hampshire.

1962-1965. Professor of Economics at Whittemore School of Business and Economics of the University of New Hampshire.

1965. Retired.

October, 1983. Died in Newtown, Pennsylvania.

Categories
M.I.T. Suggested Reading Syllabus

M.I.T. Open Economy Macroeconomics. Syllabus and bibliography. Dornbusch, 2000.

 

A genuine regret from my graduate school education was that I had not taken a course with Rudiger Dornbusch. Ex post it became clear that I would have been personally better served not having taken economic history at M.I.T. (all told, I had four semesters of solid economic history courses as an undergraduate at Yale), but unfortunately nobody shared with me the tip to take a trip on this rising star of open economy macroeconomics who began teaching at M.I.T. in 1975.

Rudi Dornbusch inspired many a classmate of mine at M.I.T. and I am sure many an economist beyond. (c.v. at M.I.T.)

_______________

14. 582 — OPEN ECONOMY MACROECONOMICS

R. Dornbusch 
Spring 2000

COURSE OUTLINE:

I.    Some Monetary History

II.    The Basic Open Economy Model

Gold Standard and the Classical Open Economy 
The Dependent Economy Model

III.    Nominal Rigidities and Policy.

Money, Wages, and Employment. 
PPP and Real Exchange Rates 
Pricing in the Open Economy

IV.    Intertemporal Trade.

V.    Finance, Exchange Rates, and Macro Implications

Capital Mobility, Exchange Rates, and Employment. 
Portfolio Diversification and Risk Premia

VI.    Policy Issues

Target Zones 
Currency Boards 
Stabilization and Exchange Rate Management 
Global Capital Markets 
Feldstein-Horioka and Capital Mobility 
Balance of Payments Crises 
Vulnerability, Crises, and Contagion 
EMU and New International Financial Architecture


TEXTS AND SOURCES:

Required readings are denoted with two asterisks (**), recommended readings with one asterisk (*).

** Grossman, G. and K. Rogoff (eds.). Handbook of International Economics. Vol. III, North Holland, 1995.
** Obstfeld, M. and K. Rogoff. Foundations of International Macroeconomics, MIT Press, 1996
Obstfeld, M. and K. Rogoff. Foundations of International Macroeconomics Workbook, MIT Press, 1998.
* Dornbusch, R. Open Economy Macroeconomics, Basic Books, 1980.
* Dornbusch, R. Exchange Rates and Inflation, MIT Press, 1988.
** See also the IMF website (http://www.imf.org) or recent IMF research reports.


Several journals are available online through JSTOR, including The Journal of Money, Credit, and Banking, The Journal of Economic Perspectives, The Journal of Political Economy, The Quarterly Journal of Economics, and The American Economic Review. Please visit JSTOR’s main page if you experience difficulty connecting to any articles from these journals to determine whether you qualify to use their site.


I.  Some Monetary History.

  • Bordo, M. and Capie. Monetary Regimes in Transition. Cambridge University Press, 1994.
  • Bordo, M. and A.Schwartz (eds.). A Retrospective on the Classical Gold Standard 1821- 1931. University of Chicago Press and NBER, 1984.
  • Cassel, G.  Money and Foreign Exchange After 1914, Constable, 1922.
  • Eichengreen, B. (ed) The Gold Standard in Theory and History, Methenu, 1985, Chapter 1.
  • DeGrauwe, P. International Money Post War Trends and Theories. Clarendon Press 198x.
  • Friedman, M.  Money Mischief. Harcourt Brace Hovanovich, 1992.
  • Kindleberger C.  Manias, Panics and Crashes : A History of Financial Crises. Basic Books 1989.
  • Kindleberger C. A.  Financial History of Western Europe. Allen & Unwin 1984.
  • Obstfeld, M. and A.M. Taylor.  Global Capital Markets: Integration, Crises and Growth, unpublished manuscript.
  • Taussig, F.  International Trade, Macmillan,1928. Parts II and III.
  • Yeager, L.  International Monetary Relations, Harper & Row, 1966 Part II.

II. The Basic Open Economy Model.

Gold Standard and The Classical Open Economy

  • ** Calvo, G. “Devaluation: Level versus Rates” Journal of International Economics, Vol ll, No. 2, Pages 165-172, May 1981.
  • * Dornbusch, R. and Mussa, M. “Consumption, Real Balances and the Hoarding Function”, International Economic Review, June 1975. Also in Exchange Rates and Inflation, Chapter 18.
  • ** Dornbusch, R., S. Fischer and P. Samuelson.“Comparative Advantage, Trade and Payments in a Ricardian Model with a Continuum of GoodsAmerican Economic Review Dec 1977. Also in Exchange Rates and Inflation, Chapter 14.
  • Dornbusch, R.  Open Economy Macroeconomics. Chapter 7.

The Dependent Economy Model

  • Calvo, G. and Rodriguez, C. “A Model of Exchange Rate Determination Under Currency Substitution and Rational Expectations”, Journal of Political Economy, No. 3, 1977, pgs 617-625.
  • * Dornbusch, R. “Real and Monetary Aspects of the Effects of Exchange Rate Changes” in Exchange Rates and Inflation, MIT Press 1988, Chapter 3.
  • * Dornbusch, R.  Open Economy Macroeconomics. Chapter 6.
  • ** Mundell, R. A. Monetary Theory, chapt. 10.
  • Salter,W. “Internal and External Balance” Economic Record, 1960.
  • Swan, T. “Economic Control in a Dependent Economy” Economic Record, 1960.

III. Nominal Rigidities and Policy.

Money, Wages and Employment

  • Brock, P. “Investment, the Current Account and the Relative price of Nontraded Goods in a Small Open Economy.” Journal of International Economics, May 1988.
  • ** Calvo, G. “Staggered Contracts and Exchange Rate Policy” in J.Frenkel (ed.) Exchange Rates and International Macroeconomics. University of Chicago Press and NBER, 1983.
  • ** Dornbusch, R.  Open Economy Macroeconomics, chapter 9.
  • * Dornbusch, R. “PPP Exchange Rate Rules and Macroeconomic Stability” In Exchange Rates and Inflation, Chapter 11.
  • Dornbusch, R. “Real Exchange Rates and Macroeconomics: A Selective Survey” Scandinavian Journal of Economics 91: (2), 1989.
  • Kouri, P. “Profitability and Growth in a Small Open Economy” in A. Lindbeck(ed.) Inflation and Employment in Open Economies, North Holland 1979.
  • * Krugman, P. and Taylor,L. “The Contractionary Effects of Devaluation”, Journal of International Economics, August 1978 pp. 445-456.
  • Lizondo, S. and P. Montiel. “Contractionary Devaluation in Developing Countries. An Analytical Survey.” International Monetary Fund Staff Papers, March 1989.
  • ** Obstfeld, M. and Rogoff, K. “Sticky-Price Models of Output, the Exchange Rate, and the Current Account” In Foundations of International Macroeconomics, Ch.9.
  • Rodriguez, C. “A Stylized Model of the Devaluation-Inflation Spiral.” International Monetary Fund Staff Papers, March 1978.
  • Dornbusch, R.  The Effectiveness of Exchange-Rate Changes, Oxford Review of Economic Policy, Vol. 12, No. 3. Pgs. 26-38.
  • ** Dornbusch, R. “The Latin Triangle

PPP and Real Exchange Rates.

  • Keynes, J.M.  A Tract on Monetary Reform, MacMillan, 1923, chapt. 3.
  • Dornbusch, R. “Purchasing Power Parity” In Exchange Rates and Inflation,Chapter 5.
  • * Frankel, J. and A. Rose. “A Panel Project on Purchasing Power Parity: Mean Reversion Within and Between Countries”Journal of International Economics,40(1) 209-224, 1996.
  • Marston, R. “Real Exchange Rates and Productivity Growth in the United States and Japan” in S. Arndt and D. Richardson (eds.) Real-Financial Linkages Among Open Economies, Cambridge: MIT Press.
  • Obstfeld and Rogoff, K. “Perspectives on PPP and Long-Run Real Exchange Rates” chapter 32 in Grossman G. and K. Rogoff (eds.) Handbook of International Economics, Vol. III., North Holland, 1995. 

Pricing in the Open Economy.

  • Dornbusch, R. “Exchange Rates and Prices” American Economic Review, March 1987. Also in Exchange Rates and Inflation, Chapter 5.
  • ** Frankel, J. and A. Rose “Empirical Research on Nominal Exchange Rates.” chapter 33 in Grossman G. and K. Rogoff (eds.) Handbook of International Economics, Vol. III., North Holland, 1995.
  • * Goldberg P.K. and M. Knetter “Goods Prices and Exchange Rates: What have we learned?” Journal of Economics Literature, vol. XXXV, September 1997, pp.1243-1272.

IV. Intertemporal Models Of The Open Economy

  • Obstfeld, M. and Rogoff,K.  Foundations of International Macroeconomics, parts 1-4.
  • * Obstfeld, M. and Rogoff, K. “The Intertemporal Approach to the Current Account” chapter 34 in Grossman G. and K. Rogoff (eds.) Handbook of International Economics, Vol. III., North Holland, 1995.
  • Fisher, E. O’N and K. Kasa.  Generational Accounting in Open Economies in FRBSF Economic Review, 1997, Number 3.
  • * Dornbusch, R. “Real Interest Rates, Home Goods and Optimal External Borrowing, Journal of Political Economy, Feb 1983 (also in Exchange Rates and Inflation, Chapter 16.)
  • ** Dornbusch, R. Notes  on Intertemporal Trade”.

V. Finance, Exchange Rates and Macro Implications 

Capital Mobility, Exchange Rates, and Employment.

  • * Dornbusch, R. “Expectations and Exchange Rate Dynamics”, Journal of Political Economy, Dec. 1976 (also in Exchange Rates and Inflation, chapter 4)
  • Frankel, J. and K. Froot. “Using Survey Data to Test Propositions Regarding Exchange Rate Expectations.” American Economic Review Vol. 77, 1987, pp. 133-153.
  • Frenkel, J. and Mussa, M. “Asset Markets, Exchange Rates and the Balance of Payments” in R. Jones and P. Kenen (eds.) Handbook of International Economics, Vol.2, North-Holland 1985.
  • * Krugman,P.  Exchange Rate Instability, MIT Press, 1988.
  • Taylor M. “The Economics of Exchange Rate”.Journal of Economic Literature, Vol.XXXIII, March 1995, pp.13-47.
  • Mundell, R.  International Economics, chapts. 14 -18.
  • De Long, J. Bradford, Shleifer A. , Summers L. , Vishny R.  “Noise Trader Risk in Financial Markets” Journal of Political Economy; V98 n.4 August 1990, pp. 703-38.
  • Frankel, Jeffrey A.; Galli, Giampaolo; Giovannini, Alberto, Editors.  “The Microstructure of Foreign Exchange Markets”.  National Bureau of Economic Research Conference Report Series. Chicago and London: University of Chicago Press, 1996.

Portfolio Diversification and Risk Premia.

  • * Dornbusch, R. “Exchange Rate Risk and the Macroeconomics of Exchange Rate Determination, in Exchange Rates and Inflation, Chapter 7.
  • Frankel, J. and Froot, K. “Forward Discount Bias: Is it an Exchange Risk Premium?”Quarterly Journal of Economics 104:1, 139-162.
  • ** Froot, K. and R. Thaler. “Anomalies: Foreign Exchange”,Journal of Economics Perspectives, vol 4, No. 3, Summer 1990, Pages l79-l92.
  • Krugman, P. “Consumption Preferences, Asset Demands, and Distribution Effects in International Financial Markets.”NBER Working Paper No. 651, March 1981.
  • Mussa,M. “Official Intervention and Exchange Rate Dynamics” in J.Bhandari (ed.) Exchange Rate Management Under Uncertainty,MIT Press 1985.
  • Sachs, J. and Wyplosz, C. “Real Exchange Rate Effects of Fiscal Policy.” NBER Working PaperNo. 1256, Jan. 1984.
  • Neely, Christopher J. “Technical Analysis and the Profitability of U.S. Foreign Exchange Intervention”.  Federal Reserve Bank of St. Louis, July/August 1998.
  • Solnik, Bruno.  “Global Asset Management: Too Hedge or Not to Hedge–a Question That Cannot be Ignored”.  The Journal of Portfolio Management, Summer 1998.
  • Stein, Jerome L. and G. Paladino.  Recent Developments in International Finance: A Guide to Research Journal of Banking and Finance,21 (1998) 1685-1720.
  • Stulz, Rene M.  “International Portfolio Choice and Asset Pricing: An Integrative Survey”, in R. Jarrow et al (eds)Handbooks in OR & MS, vol. 9, Elsevier Science B.V., 1995.

VI. Policy Issues

Target Zones

  • Bertola, G. “Continuos-Time Models of Exchange Rates and Intervention ” In F. van der Ploeg, ed.Handbook of International Macroeconomics, Blackwell, 1994.
  • Bertola, G. and Caballero, R.“Target Zones and Realignments”American Economic Review, 1992.
  • ** Krugman, P. and M. Miller. (eds.) Exchange Rate Targets and Currency Bands. Cambridge University Press, 1992.
  • ** Svensson, L. “An Interpretation of Recent Research on Exchange Rate Target Zones.”Journal of Economics Perspectives, Vol 6, No. 4, pp. 119-144, Fall 1992.
  • Werner, A. “Exchange Rates and Target Zone Width”. Economic Letters. Dec 1992.

Exchange Rate Regimes and Currency Boards

  • Calvo, G. “On Dollarization”,1999 (www.bsos.umd.edu.econ.calvo.htm)
  • Dornbusch R. and F. Giavazzi,“Hard Currency and Sound Credit: A Financial Agenda for Central Europe” May 1998.
  • Eichengreen, B.“The Choice of Exchanage Rate Regime” (incomplete).
  • Eichengreen, B.  Golden Fetters, Oxford University Press, 1992.
  • Friedman, M. “The Case for Flexible Exchange Rates” in his Essays in Positive Economics. U. of Chicago Press, 1953.
  • **Goldstein, M. Exchange Rate System and the IMF. Institute for International Economics, 1995
  • Hanke, S. and K. Schuler  Currency Boards for Developing Countries. ICS Press, 1994.
  • List of References to Currency Boards by C. Broda.
  • Mihalke D. “Currency Board Arrangements: Issues and Experiences”.International Monetary Fund Occasional Paper No. 151, 1997.
  • Mundell, R. “A Theory of Optimum Currency Areas”, American Economic Review 51, September 1961.
  • Obstfeld, M. and K. Rogoff, “The Mirage of Fixed Exchange Rates” Journal of Economic Perspectives 9, Fall 1995
  • Perry G. E. “Currency Boards and External Shocks: How Much Pain, How Much Gain?”, World Bank Latin American and Caribbean Studies, February 1997.
  • Taussig, F. International Trade, Macmillan, 1928.  Parts II and III.
  • Tornell, A. and A. Velasco, “Fixed vs. Flexible Exchange Rates: Which Provides More Fiscal Discipline?”NBER WP. No. 5108, 1995.
  • * Williamson, J. What Role for Currency Boards? Institute for International Economics, Washington D.C., Sep. 1995.
  • Ghosh, A., A. Gulde and H. Wolf.  “Currency Boards: The Ultimate Fix?” IMF, wp. 98/8
  • ** Williamson, J.  The Crawling Band as an Exchange Rate Regime: Lessons from Chile, Colombia and Israel, Institute of International Economics, 1996.

Stabilization and Exchange Rate Management

  • Alesina, A. and A. Drazen. “Why are Stabilizations Delayed?” American Economic Review, December 1991.
  • Bruno, M. et. al.  Lessons of Economic Stabilization and its Aftermath.MIT Press, 1991.
  • Calvo, G. “Incredible Reforms” in G.Calvo (ed.) Debt, Stabilization and Development. Oxford: Basil Blackwell, 1989.
  • Caballero, R. Structural Volatility In Modern Latin America, April, 2000
  • Calvo, G. and C. Vegh. “Credibility and the Dynamics of Stabilization Policy: A Basic Framework.” IMF, Staff Papers, November 1990.
  • * Dornbusch, R. “Credibility and Stabilization”, Quarterly Journal of Economics, August 1991, pp. 837-850. Also in R.Dornbusch Stabilization, Debt, and Reform, Prentice Hall 1993.
  • * “The New Classical Macroeconomics and Stabilization Policy.”American Economic Review, May 1990. Also in R. Dornbusch Stabilization, Debt, and Reform, Prentice Hall 1993.
  • Dornbusch, R., and S. Edwards, S (eds)The Macroeconomics of Populism in Latin America. University of Chicago Press, 1991.
  • Dornbusch, R. and Fischer, S. “Stopping Moderate Inflations.” World Bank Review, 1992.
  • Dornbusch, R., Goldfajn, I. and Valdes, R. “Currency Crisis and Collapses”, Brookings Papers, 2, 1995.
  • Drazen, A. and Helpman, E. “Stabilization With Exchange Rate Management” Quarterly Journal of Economics, 1987.
  • Fernandez, R. and D. Rodrik. “Resistance to Reform. Status Quo Bias in the Presence of Individual Specific Uncertainty” American Economic Review, December 1991.
  • Goldfajn I. and R. O. Valdes“The Aftermath of Appreciations”. NBER WP#5650, July 1996.
  • ** Vegh, C. “Stopping High Inflation: An Analytical Overview,” International Monetary Fund, Staff Papers, September 1992.
  • * Williamson, J. “The Crawling Band as an Exchange Rate Regime: Lessons from Chile, Colombia and Israel”, Institute of International Economics,1996.

Feldstein-Horioka and Long Term Capital Mobility.

  • Cardoso, E. and R. Dornbusch. “Foreign Private Capital Flows” Handbook of Development Economics, Vol II, North Holland, 1989.
  • **Dooley, M., J. Frankel and D. Mathieson “International Capital Mobility: What do Saving-Investment Correlations Tell US?” International Monetary Fund, Staff Papers, September 1987.
  • ** Feldstein, M. and Horioka, C. “Domestic Savings and International Capital Flows”. Economic Journal, June 1980, pages 314-329.
  • Frankel, J. “Measuring International Capital Mobility: A Review.”American Economic Review. Papers and Proceedings. May 1992.
  • Harberger, A.“Vignettes on the World Capital Market”American Economic Review, May 1980.
  • ** Ventura, J. & A. Kraay, “Current Accounts in Debtor and Creditor Countries”, March 1999. ECONOMETRICS SEMINAR (Joint Harvard/MIT) 

Balance of Payments Crises.

  • Calvo, G. “Balance of Payments Crises in a Cash in Advance Economy”, Journal of Money, Credit and Banking,1987.
  • Dornbusch, R. “Collapsing Exchange Rate Regime”, Journal of Development Economics, Vol 27, No 1-2, Pgs 7l-83, October 1987.
  • Flood, R. and P.Garber, 1984, “Collapsing Exchange Rate Regimes: Some Linear Examples.” Journal of International Economics,1/2, 1-13.
  • Froot K and M. Obstfeld, “Exchange Rate Dynamics Under Stochastic Regime Shifts: A  Unified Approach”, NBER WPNo. 2835, February 1989.
  • ** Garber, P. and L. Svensson “The Operation and Collapse of Fixed Exchange Rate Regimes” chapter 36, section 3, in Grossman G. and K. Rogoff (eds.) Handbook of International Economics,Vol. III., North Holland, 1995.
  • ** Krugman, P. “A Model of Balance of Payments Crises.”Journal of Money, Credit and Banking, 3, 1979 ,311-325.
  • ** Salant,S. and D. Henderson, 1978,”Market Anticipations of Government Policies and the Price of Gold” Journal of Political Economy, 4, 627-648.

Crises and Contagion

  • Caballero, R. and Krishnamurthy, “Emerging Markets Crises: An Asset Market Perspective”, MIT mimeo, 1998.
  • Calvo, Guillermo A.”Capital FLows and Capital-Market Crises: The Simple Economics of Sudden Stops”, unpublished manuscript, U. of Maryland, 1998 (http://www.bsos.umd.edu/econ/ciecalvo.htm)
  • Calvo, Guillermo A. “Understanding the Russian Virus”, unpublished manuscript, U. of Maryland, 1998 (http://www.bsos.umd.edu/econ/calvo.htm)
  • Chang and Velasco, “Financial Crises in Emerging Markets: A Canonical Model” NBER WP No. 6469, 1997.
  • Diaz Alejandro, C. “Good-bye Financial Repression, Hello Financial Crisis” Journal of Development Economics, 19, 1985.
  • Dornbusch,R. “Asian Themes”, (http://web.mit.eud/rudi/www)  Feb. 1998.
  • Dornbusch,R. “Brazil’s Incomplete Stabilization and Reform”, Brookings Papers on Economic Activity 1:1997, pp.367-404 (http://web.mit.eud/rudi/www.)
  • Eichengreen B., A. Rose, C. Wyplosz. “Contagious Currency Crises”. Scandinavian Journal of Economics,December 1996.
  • Goldfajn, I. and Valdes, “Capital Flows and the Twin Crisis: The Role of Liquidity” IMF WP 97/87.
  • IMF, World Economic Outlook, Fall 1998 (http://www.imf.org)
  • IMF, International Capital Markets, Fall 1998 (http://www.imf.org)
  • *Kaminsky G. and C.Reinhart, “On Crises, Contagion and Confusion.” Unpublished manuscript, George Washington University and U. of Maryland, 1998.
  • Kaminsky, G. “Leading Indicators of Currency Crises” Board of Governors of the Federal Reserve, mimeo.  Board of Governors of the Federal Reserve, December, 1997.
  • Krugman P. “The Myth of Asia’s Miracle”. Foreign Affairs, November/December 1994.
  • Krugman PWhat happened to Asia?” January 1998 (http://web.mit.edu/krugman/www)
  • Obstfeld, M., “The Logic of Currency Crises”, Cahiers Economique and Monetaires, 43, 1995.
  • ** Tobin, J.  “A Proposal for International Monetary Reform” in his Essays in Economics, MIT Press, 1982, chapt. 20.

EMU and New International Financial Architecture

  • Dornbusch,R., C.Favero and F. Giavazzi “Challenges for the European Central Bank” Economic Policy, April 1998, pp. 15-64, CEPR (http://web.mit.edu/rudi/www).
  • Eichengreen, B. “Toward a New International Financial Architecture: A Practical Post-Asia Agende”, IIE Press, 1999.
  • Eichengreen, B.”European Monetary Unification: Theory, Practice, Analysis”, MIT Press, 1997.
  • Fischer, S. “Reforming the International Monetary System” David Finch lecture, November, 1998 (http://www.imf.org/external/np/speeches/1998/110998.HTM).
  • Fischer, S. “On the Need for an International Lender of Last Resort” New York, January 3, 1999.
  • McCaughley, “The Euro and the Dollar”, Princeton Essays in International Finance, 1997

Source: August 17, 2000 webpage capture by Internet Archive Wayback Machine.

Image Source: Rudiger Dornbusch from the website of the MIT Museum.

Categories
Economists Michigan

Michigan. Economics Faculty List from Centennial Symposium, 1980

 

The Michigan economics department celebrated its Centennial in 1980 and appended a list of present and former faculty to the symposium program, a copy of which found its way to Wolfgang Stolper’s papers, now at Duke. 

This post provides chronological and alphabetical lists of economics faculty (for the rank of lecturer and above, with a minimum three years of service) at the University of Michigan for 1880-1980.

A brief history of the University of Michigan’s economic department through 1940 has been posted earlier.

__________________

Those listed were in the department three years or more at a rank of Lecturer and up. The concluding year for faculty who moved into the Business School is shown as 1924; for Sociology as 1929. Current [1980] faculty members’ names appear in all capital letters.

Chronological List

Adams, Henry Carter 1880-1921
Cooley, Charles H. 1892-1929
Dixon, Frank H. 1892-1898
Taylor, Fred M. 1892-1929
Jones, Edward D. 1901-1919
Smalley, Harrison S. 1903-1912
Friday, David 1908-1921
Parry, Carl E. 1908-1912
Hamilton, Stuart M. C. 1910-1914
Dowrie, George W. 1913-1918
Rottschaffer, Henry 1913-1916
Sharfman, I. Leo 1913-1954
Thompson, Warren S. 1913-1918
Ivey, Paul W. 1914-1917
Kolbe, Frank F. 1914-1917
Rodkey, Robert G. 1914-1917
Paton, William A. 1915, 1917-1959
Tucker, Rufus S. 1915-1919
Calhoun, Wilbur P. 1916-1923
Leffler, Roy V. 1916-1919
Schmitt, Herbert N. 1916-1923
Kilborn, Russell D. 1917-1920
Wood, Arthur E. 1917-1929
Holmes, Roy N. 1918-1930
Caverly, Harcourt L. 1919-1933
Griffen, Clare E. 1919-1924
Walker, Ross G. 1919-1922
Cahow, Paul D. 1920-1924
Edmonds, Charles C. 1920-1925
Ellis, Howard S. 1920-1922, 1925-1938
Lubin, Isador 1920-1923
May, Carroll 1920-1930
Ross, Francis E. 1920-1927
Carr, Lowell J. 1921-1929
Horner, Seward L. 1921-1926
Oppenheim, Saul C. 1921-1926
Peterson, Shorey 1921-1967
Wyngaarden, Herman 1921-1924
Angell, Robert C. 1922-1929
Day, Edmund E. 1922-1927
Lewis, Ben W. 1922-1925
Mason, Perry 1922-1930
Watkins, Leonard L. 1922-1924, 1926-1957
Bigge, George E. 1923-1927
Dickinson, Z. Clark 1923-1959
Devol, Floyd E. 1924-1936
Elliot, Margaret 1924-1949
Foscue, Augustus W. 1924-1927
Goodrich, Carter L. 1924-1931
Selbey, Harold K. 1924-1928
Sickle, John V. Van 1924-1928
Whitlow, Claude J. 1925-1929
Woodworth, G. Walter 1925-1930
Adams, Leonard W. 1926-1929
Engle, Nathanael H. 1926-1930
Briggs, Robert P. 1927-1945
Horton, Donald C. 1927-1935
Remer, Charles F. 1928-1959
Crandell, William T. 1929-1935
Horner, Robert R. 1929-1941
Robinson, Roland J. 1929-1935
Copeland, Morris A. 1930-1936
Hoad, William M. 1930-1934
Laing, Lemuel L. 1930-1945
Palmer, William B. 1930-1976
Timoshenko, Vladimir 1930-1935
Handman, Max S. 1931-1940
Ford, Robert S. 1934-1968
Haber, William 1936-1968
Hoover, Edgar M. 1936-1947
Simmons, Edward C. 1936-1947
Smithies, Arthur 1938-1946
ACKLEY, GARDNER 1940-
Anderson, George R. 1942-1972
Bond, Floyd A. 1942-1946
Wixon, Rufus 1944-1947
Patterson, Gardner 1947-1950
Katona, George 1947-1972
LEVINSON, HAROLD M. 1947-
Musgrave, Richard A. 1947-1959
Suits, Daniel B. 1947-1970
Boulding, Kenneth E. 1949-1968
Smith, Warren L. 1949-1954, 1957-1972
STOLPER, WOLFGANG F. 1949-
Klein, Lawrence R. 1950-54
MORGAN, JAMES N. 1950-
Brower, Tony 1952-1955
Lansing, John B. 1956-1970
Rousseas, Stephen W. 1956-1959
BRAZER, HARVEY E. 1957-
Fletcher, Daniel O. 1957-1960
Hutchinson, Harry D. 1957-1960
MUELLER, EVA L. 1957-
Winger, William P. 1957-1960
BORNSTEIN, MORRIS 1958-
Shearer, Ronald A. 1958-1962
Hayes, Samuel P. 1959-1962
ANDERSON, W. H. LOCKE 1960-
Ayal, Eliezer B. 1960-1963
FUSFELD, DANIEL R. 1960-
Milstein, David N. 1960-1963
BARLOW, ROBIN 1961-
Eckstein, Alexander 1961-1976
STERN, ROBERT M. 1961-
Babcock, Jarvis M. 1962-1966
Butter, Irene 1962-1966
Chao, Kang 1962-1965
Shulman, Mary Alice 1962-1975
TEIGEN, RONALD L. 1962-
Morss, Elliott R. 1963-1965
Parker, John E. 1963-1967
SHEPHERD, WILLIAM G. 1963-
Tilly, Richard 1963-1966
HYMANS, SAUL H. 1964-
PORTER, RICHARD C. 1964-
SHAPIRO, HAROLD T. 1964-
CROSS, JOHN G. 1965-
HOLBROOK, ROBERT S. 1965-
BERG, ELLIOT J. 1966-
Demeny, Paul 1966-1969
JOHNSON, GEORGE E. 1966-
Munk, Bernard 1966-1969
STAFFORD, FRANK P. 1966-
Yoshihara, Kunio 1966-1969
CRAFTON, HELEN P. 1967-
Eckstein, Peter C. 1967-1971
NEENAN, WILLIAM B. 1967-
Scherer, Frederic M. 1967-1973
Cohen, Malcolm 1968-1973
DERNBERGER, ROBERT F. 1968-
FELDSTEIN, PAUL J. 1968-
FREEDMAN, DEBORAH S. 1968-
Klass, Michael W. 1968-1974
STEINER, PETER O. 1968-
Strumpel, Burkhard 1968-1975
Winter, Sidney G. 1968-1976
Hill, C. Russell 1969-1975
Manove, Michael E. 1969-1975
Simmons, George B. 1969-1974
Taylor, Lester D. 1969-1974
DEARDORFF, ALAN V. 1970-
Lee, Ronald D. 1970-1979
SAXONHOUSE, GARY R. 1970-
Shoup, Donald C. 1970-1974
Heller, Peter 1971-1977
Roistacher, Elizabeth 1972-1975
RUBINFELD, DANIEL L. 1972-
WEISSKOPF, THOMAS L. 1972-
WRIGHT, GAVIN 1972-
COURANT, PAUL N. 1973-
HOWREY, E. PHILIP 1973-
JUSTER, F. THOMAS 1973-
KMENTA, JAN 1973-
ADAMS, WILLIAM JAMES 1974-
BERGSTROM, THEODORE C. 1975-
DUNCAN, GREG. J. 1975-
LAITNER, JOHN P. 1975-
ANDERSON, ANN P. 1976-
CURTIN, RICHARD T. 1976-
GRAMLICH, EDWARD M. 1976-
BLUME, LAWRENCE 1977-
SIMON, CARL P. 1977-
VARIAN, HAL R. 1977-
RANNEY, SUSAN I. 1978-
WEBB, STEVEN B. 1978-
LOURY, GLENN C. 1979-

 

Alphabetical list

ACKLEY, GARDNER 1940-
Adams, Henry Carter 1880-1921
Adams, Leonard W. 1926-1929
ADAMS, WILLIAM JAMES 1974-
ANDERSON, ANN P. 1976-
Anderson, George R. 1942-1972
ANDERSON, W. H. LOCKE 1960-
Angell, Robert C. 1922-1929
Ayal, Eliezer B. 1960-1963
Babcock, Jarvis M. 1962-1966
BARLOW, ROBIN 1961-
BERG, ELLIOT J. 1966-
BERGSTROM, THEODORE C. 1975-
Bigge, George E. 1923-1927
BLUME, LAWRENCE 1977-
Bond, Floyd A. 1942-1946
BORNSTEIN, MORRIS 1958-
Boulding, Kenneth E. 1949-1968
BRAZER, HARVEY E. 1957-
Briggs, Robert P. 1927-1945
Brower, Tony 1952-1955
Butter, Irene 1962-1966
Cahow, Paul D. 1920-1924
Calhoun, Wilbur P. 1916-1923
Carr, Lowell J. 1921-1929
Caverly, Harcourt L. 1919-1933
Chao, Kang 1962-1965
Cohen, Malcolm 1968-1973
Cooley, Charles H. 1892-1929
Copeland, Morris A. 1930-1936
COURANT, PAUL N. 1973-
CRAFTON, HELEN P. 1967-
Crandell, William T. 1929-1935
CROSS, JOHN G. 1965-
CURTIN, RICHARD T. 1976-
Day, Edmund E. 1922-1927
DEARDORFF, ALAN V. 1970-
Demeny, Paul 1966-1969
DERNBERGER, ROBERT F. 1968-
Devol, Floyd E. 1924-1936
Dickinson, Z. Clark 1923-1959
Dixon, Frank H. 1892-1898
Dowrie, George W. 1913-1918
DUNCAN, GREG. J. 1975-
Eckstein, Alexander 1961-1976
Eckstein, Peter C. 1967-1971
Edmonds, Charles C. 1920-1925
Elliot, Margaret 1924-1949
Ellis, Howard S. 1920-1922, 1925-1938
Engle, Nathanael H. 1926-1930
FELDSTEIN, PAUL J. 1968-
Fletcher, Daniel O. 1957-1960
Ford, Robert S. 1934-1968
Foscue, Augustus W. 1924-1927
FREEDMAN, DEBORAH S. 1968-
Friday, David 1908-1921
FUSFELD, DANIEL R. 1960-
Goodrich, Carter L. 1924-1931
GRAMLICH, EDWARD M. 1976-
Griffen, Clare E. 1919-1924
Haber, William 1936-1968
Hamilton, Stuart M. C. 1910-1914
Handman, Max S. 1931-1940
Hayes, Samuel P. 1959-1962
Heller, Peter 1971-1977
Hill, C. Russell 1969-1975
Hoad, William M. 1930-1934
HOLBROOK, ROBERT S. 1965-
Holmes, Roy N. 1918-1930
Hoover, Edgar M. 1936-1947
Horner, Robert R. 1929-1941
Horner, Seward L. 1921-1926
Horton, Donald C. 1927-1935
HOWREY, E. PHILIP 1973-
Hutchinson, Harry D. 1957-1960
HYMANS, SAUL H. 1964-
Ivey, Paul W. 1914-1917
JOHNSON, GEORGE E. 1966-
Jones, Edward D. 1901-1919
JUSTER, F. THOMAS 1973-
Katona, George 1947-1972
Kilborn, Russell D. 1917-1920
Klass, Michael W. 1968-1974
Klein, Lawrence R. 1950-54
KMENTA, JAN 1973-
Kolbe, Frank F. 1914-1917
Laing, Lemuel L. 1930-1945
LAITNER, JOHN P. 1975-
Lansing, John B. 1956-1970
Lee, Ronald D. 1970-1979
Leffler, Roy V. 1916-1919
LEVINSON, HAROLD M. 1947-
Lewis, Ben W. 1922-1925
LOURY, GLENN C. 1979-
Lubin, Isador 1920-1923
Manove, Michael E. 1969-1975
Mason, Perry 1922-1930
May, Carroll 1920-1930
Milstein, David N. 1960-1963
MORGAN, JAMES N. 1950-
Morss, Elliott R. 1963-1965
MUELLER, EVA L. 1957-
Munk, Bernard 1966-1969
Musgrave, Richard A. 1947-1959
NEENAN, WILLIAM B. 1967-
Oppenheim, Saul C. 1921-1926
Palmer, William B. 1930-1976
Parker, John E. 1963-1967
Parry, Carl E. 1908-1912
Paton, William A. 1915, 1917-1959
Patterson, Gardner 1947-1950
Peterson, Shorey 1921-1967
PORTER, RICHARD C. 1964-
RANNEY, SUSAN I. 1978-
Remer, Charles F. 1928-1959
Robinson, Roland J. 1929-1935
Rodkey, Robert G. 1914-1917
Roistacher, Elizabeth 1972-1975
Ross, Francis E. 1920-1927
Rottschaffer, Henry 1913-1916
Rousseas, Stephen W. 1956-1959
RUBINFELD, DANIEL L. 1972-
SAXONHOUSE, GARY R. 1970-
Scherer, Frederic M. 1967-1973
Schmitt, Herbert N. 1916-1923
Selbey, Harold K. 1924-1928
SHAPIRO, HAROLD T. 1964-
Sharfman, I. Leo 1913-1954
Shearer, Ronald A. 1958-1962
SHEPHERD, WILLIAM G. 1963-
Shoup, Donald C. 1970-1974
Shulman, Mary Alice 1962-1975
Sickle, John V. Van 1924-1928
Simmons, Edward C. 1936-1947
Simmons, George B. 1969-1974
SIMON, CARL P. 1977-
Smalley, Harrison S. 1903-1912
Smith, Warren L. 1949-1954, 1957-1972
Smithies, Arthur 1938-1946
STAFFORD, FRANK P. 1966-
STEINER, PETER O. 1968-
STERN, ROBERT M. 1961-
STOLPER, WOLFGANG F. 1949-
Strumpel, Burkhard 1968-1975
Suits, Daniel B. 1947-1970
Taylor, Fred M. 1892-1929
Taylor, Lester D. 1969-1974
TEIGEN, RONALD L. 1962-
Thompson, Warren S. 1913-1918
Tilly, Richard 1963-1966
Timoshenko, Vladimir 1930-1935
Tucker, Rufus S. 1915-1919
VARIAN, HAL R. 1977-
Walker, Ross G. 1919-1922
Watkins, Leonard L. 1922-1924, 1926-1957
WEBB, STEVEN B. 1978-
WEISSKOPF, THOMAS L. 1972-
Whitlow, Claude J. 1925-1929
Winger, William P. 1957-1960
Winter, Sidney G. 1968-1976
Wixon, Rufus 1944-1947
Wood, Arthur E. 1917-1929
Woodworth, G. Walter 1925-1930
WRIGHT, GAVIN 1972-
Wyngaarden, Herman 1921-1924
Yoshihara, Kunio 1966-1969

 

Source: University of Michigan, Department of Economics. Centennial Celebration and Symposium (Ann Arbor, Michigan: April 11-12, 1980), pp. 9-11. Found in Duke University. David M. Rubenstein Rare Book and Manuscript Library. Economists’ Papers Archive. Papers of Wolfgang Stolper, Box 9.

Image Source:  Door to the Economics Building. Clipped from larger photograph of the building in April 1952, University of Michigan, Bentley Historical Library. Incidentally the building was destroyed by an arson fire on Christmas Eve, 1981.

Categories
Exam Questions Harvard Methodology

Harvard. Final Exam for Scope and Method of Economics. Taussig, 1896.

 

Frank Taussig returned from a sabbatical to teach a course on the scope and method(s) of economics at Harvard during the second term of 1895-96. The following years his colleague, the economic historian William Ashley, taught the course.

The enrollment figures and final examination questions for Taussig’s course are provided below.

____________________

COURSE ANNOUNCEMENT

[Economics] 13hf. Scope and Method in Economic Theory and Investigation. Half-course. Wed., Fri., and (at the pleasure of the instructor) Mon., at 1.30 (second half-year). Professor Taussig.

Source: The Harvard University Catalogue, 1895-96,p. 100.

 

COURSE ENROLLMENT

[Economics] 132. Professor Taussig.—Scope and Method in Economic Theory and Investigation. hf. 2 hours, 2d half-year.

Total 14: 11 Graduates, 3 Seniors.

 

Source: Harvard University. Report of the President of Harvard College, 1895-1896, p. 63.

 

1895-96
ECONOMICS 13.
[Final examination]

  1. Compare Wagner’s enumeration of the problems within the scope of economic science with Keynes’s; and consider what doubts or objections there may be in regard to any of the problems mentioned by either writer.
  2. Explain and examine critically one of the following passages in Wagner:

Section 63 (pp. 158-163).
Section 70 (pp. 180-182).

  1. Illustrate the mode in which use is advantageously made of the deductive and the inductive method in regard to two of the following topics:

the causes which determine the general range of prices;
the prospects of socialism;
the prospects of cooperation.

  1. What peculiarities and difficulties appear for economic science in the choice of terminology and in definition? Illustrate.
  2. Is there ground for saying that the economic history of very recent times is of greater value for economic theory than the economic history of remote periods?
  3. What do you conceive to be the position in regard to method in economics of Ricardo? J.S. Mill? Roscher? Schmoller?

 

Source: Harvard University Archives. Prof. F. W. Taussig Examination Papers in Economics, 1882-1935, (HUC 7882), p. 55.

Image Source: Harvard Portfolio, vol. VI, 1895 .

Categories
Bibliography Chicago

Chicago. Course Bibliography (books). Economics and Social Institutions. Knight, 1949

 

 

Together Frank Hyneman Knight (Morton D. Hull Distinguished Service Professor of the Social Sciences) and Charner Marquis Perry (Associate Professor of Philosophy) taught a course at mid-century on institutional economics with the title “Economics and Social Institutions”. The course was a joint graduate offering of the departments of economics and philosophy at the University of Chicago. This post provides a transcription of a bibliography of books for the course that was found filed among Milton Friedman’s papers at the Hoover Institution Archives. One presumes from the title “Bibliography A: Books” that there must have been a “Bibliography B: articles and chapters”, but to find a copy of that B-Bibliography, we will need to go elsewhere and have a bit of luck.

_______________

Course Announcement

[Economics] 305. Economics and Social Institutions (identical with Philosophy 305). The relations between the classical mathematical and the institutional historical views of economic phenomena; institutional factors as the framework and much of the content of the price economy; late nineteenth-century economic society as a complex of structural forms. Prereq: Econ 301 and some European economic history. Win: M 3:30-5:30; Knight, Perry.

 

Source:  University of Chicago. Announcements, Sessions of 1950-1951. Volume L, No. 3 (June 1, 1950), p. 29.

_______________

ECONOMICS AND SOCIAL INSTITUTIONS.
(ECON. 305; Philos. 305)

BIBLIOGRAPHY—A:  BOOKS.
(WINTER, 1949)

Ardzrooni, L. (Ed.)—Essays in our Changing Order (Veblen)

Ayres, C.E.—The Theory of Economic Progress

Ibid.—The Economic Order
Ibid.—The Divine Right of Capital

Ballard, L.V.—Social Institutions

Barnes, Harry E.—History and Prospects of the Social Sciences

Ibid.—Intellectual and Cultural His. Of the Western World

Barth, Paul,—Die Philosophie der Geschichte als Sociologie

Barnes, H.E. and Becker,—Social Thought from Lore to Science

Beard, Miriam,—History of the Business Man

Bucher, Karl,–Industrial Evolution

Bury, J.B.—The idea of Progress

Ibid.—History of Freedom of Thought
Ibid.—Evolution and History (in Evolution in Modern Thought)

Clark, John M.—Essays in Social Economics

Commons, John R.—Institutional Economics

Ibid.—Legal Foundations of Capitalism

Dewey, John,–Influence of Darwin on Philosophy

Dickinson, H.D.—Institutional Revenue

Dorfman, Joseph—Thorstein Veblen and His America

Engel-Janozi, Fr.—Growth of German Historicism

Einstein, Lewis,–Historical Change

Evolution in Modern Thought, (Mod. Lib.—Various authors)

Gambs, John S.—Beyond Supply and Demand (Bibliog., short)

Gras, N.S.B.—Introduction to Economic History

Ibid.—Business and Capitalism

Gruchy, Allan L.—Modern Economic Thought; The American Contribution

Hamilton, Walton H.—The Pattern of Competition

Hayes, E.C. (Ed.)—Recent Developments in the Social Sciences (J.M. Clark)

Hertzler, J.O.—Social Institutions

Herskovits, J.M.—The Economic Life of Primitive Peoples

Homan, Paul T.—Contemporary Economic Thought

Hook, Sidney,—From Hegel to Marx

Ibid.—Toward the Understanding of Karl Marx

Huxley, Julian,—Evolution

Jones, Richard,—Introductory Lecture on Political Economy

Keynes, J.N.—Scope and Method of Pol. Econ. (Esp. Chaps. IX, X)

Korsch, Karl,—Karl Marx

Miller, Hugh,—History and Science

Mitchell, Wesley C.—The Backward Art of Spending Money, etc.

Mitchell, William,—The Early History of the Law Merchant

Morgan, C. Lloyd,—Emergent Evolution

Ibid.—The Emergence of Novelty

Mukerjee, R.—The Institutional Theory of Economics

Mumford, Lewis,—Technics and Civilization

Müller-Lyer,—A History of Social Development (Econ. Stages)

Murchison, C. (Ed.)—Psychologies of 1925

Ogburn, William F.—Social Change

Ibid., and Goldenweiser, E.A.—Social Sciences in Interrelations

Parsons, Talcott,—The Structure of Social Action

Pound, Roscoe,—Interpretations of Legal History

Rice, Stuart A. (Ed.)—Methods in Social Science

Robertson, H.M.—The Rise of Economic Individualism

Sapir, Edward—Language (Chs. 7-8 on linguistic change)

Sée, Henri,—The Economic Interpretation of History

Ibid.—Modern Capitalism (Les origins de cap. modern)

Seligman, E.R.A.—The Economic Interpretation of History

Shotwell, J.T.—Introduction to History of History (Introduction and last chapter)

Simiand, François,—La méthode positive en économie politique

Small, A.W.—The Origins of Sociology (Historism and Methodenstreit)

Sombart, Werner,—The Quintessence of Capitalism

Ibid.—Die drei Nationalökonomien; Der moderne Kapitalismus

Spengler, O.—Decline of the West

Sumner, William G.—Folkways

Tawney, R.H.—Religion and the Rise of Capitalism

Teggart, F.J.—The Theory of History

Teggart, R.V.—Thorstein Veblen

Toynbee, A.—The Study of History

Troeltsch, Ernst,—Der Historismus; Other works

Tugwell, R.G. (Ed.)—The Trend of Economics

Veblen, Thorstein B.—The Place of Science in Civilization, etc.

Ibid.—(W.C. Mitchell, Ed.)—What Veblen Taught

Ibid.—(L. Ardzrooni, Ed.)—Essays in our Changing Order

Weber, Max,—Protestantism and the Spirit of Capitalism (Tr. Parsons)

Ibid.—General Economic History (Tr. Knight)

Ibid.—Wirtschaft und Gesellschaft (Tr. in part, Parsons, Anderson)

*  *  * *  *  *

Economic History. Heaton; Knight, Barnes and Fluegel, etc.

Histories of Economic Thought, on “schools”; on the substance, esp. Edmund Whittaker, History of Economic Ideas, first 7 Chaps.

Encyclopedias, especially Encyclopaedia of the Social Sciences. Especially articles on Economics, Secs. on Historical and Institutional Schools and on Economic History; also on Institutions, etc., etc.

 

Source:  Hoover Institution Archives. Papers of Milton Friedman. Box 77, Folder 5 “University of Chicago, Econ. 305”.

Image Source: Frank H. Knight from University of Chicago Photographic Archive, apf1-03513, Special Collections Research Center, University of Chicago Library.

Categories
Chicago Exam Questions Suggested Reading Undergraduate

Chicago. Undergraduate Money and Banking. Exams, readings. Friedman, 1946-49

 

Besides teaching in the core graduate price theory course at Chicago, Milton Friedman also covered undergraduate money and banking upon joining the faculty of the economics department. Below some material transcribed from a folder of course material found in Milton Friedman’s papers at the Hoover Institution Archives. Where answers were provided to some examination questions, they have been transcribed [and placed in square brackets] and included below.

Fun Fact: According to class rolls kept by Friedman, Marc Nerlove was a student in the Autumn 1951 Money and Banking class taught by Friedman.

_________________

Course Announcement and Description

[Economics] 230. Introduction to Money and Banking. Study of factors which determine the value of money in the short and in the long run; and operation of the commercial banking system and its relation to the price level and general business activity. Prereq: Soc Sci 2 and Econ 210, or equiv. Aut: MWF 10:30 Friedman; Win: MWF 2:30; Mints.

Source:   University of Chicago. Announcements. The College and the Divisions, Sessions of 1947-1948. Vol. XLVII, No. 4 (May 15, 1947), p. 224.

_________________

Text for Economics 230:

L. V. Chandler, The Economics of Money and Banking. Harper & Brothers.

The Book will be used again as a text when the course is given in the Winter Quarter. Give the number in class as that of the Autumn, 1947.
Reserve List & Bookstore.

_________________

Economics 230
Autumn 1947
Library Book List

Robertson, D. H. Money

Gregory, T. E. The Gold Standard and Its Future (3rd)

Board of Governors. Federal Reserve System.Its Purposes and Function

_________________

Economics 230
Autumn 1951

Supplementary Readings and Problem for Reading Period

Readings

Text: Lester Chandler, Economics of Money and -Banking

  1. American Economic Association, Readings in Monetary Theory, edited by Friedrich Lutz and Lloyd W. Mints.
  2. Goldenweiser, E.A., American Monetary Policy.
  3. Gregory, T.E., The Gold Standard and Its Future.
  4. Hardy, C.O., Credit Policies of the Federal Reserve System.
  5. Keynes, J.M., Essays in Persuasion.
  6. Mints, Lloyd W., Monetary Policy for a Competitive Society.
  7. Robertson, D.H., Money.
  8. S. Board of Governors of the Federal Reserve System, The Federal Reserve System, Its Purposes and Functions.

 

Problem

            For a convenient date in 1951, estimate the maximum amount of currency and deposits that would have been outstanding if the banking system had used all the possibilities of monetary expansion available under the then existing laws and regulations about reserve requirements of member and non-member banks and about reserve requirements of Federal Reserve Banks. For purposes of the computation, assume (a) an unchanged amount of Treasury currency outstanding; (b) elimination of Treasury deposits with Federal Reserve Banks through purchase of government securities held by the Federal Reserve Banks. With respect to all other factors—such as percentage distribution of public’s money holdings among currency, demand deposits, and time deposits—you are to choose your own assumptions, the choice of reasonable assumptions and the presentation of evidence for them being an essential part of the problem.

_________________

MIDQUARTER EXAMINATION IN ECONOMICS 230
[no date, though likely 1947]

  1. Indicate the factors that principally determine—
    1. (15 points) The ratio of the amount of currency in circulation to the amount of bank deposits.
    2. (15 points) The ratio of the amount of bank deposits to the amount of reserves held by the banking system when there are no legal reserve requirements.
  2. (35 points) In country A, important new discoveries of oil are made, driving down the price of oil in that country relative to the world price. Assume that this is the only important change relevant to international trade. Trace the effects of this change on exchange rates, gold flows, price levels, imports and exports, and incomes, in country A and in the rest of the world on the assumption (a) that a strict gold standard is in operation; (b) that inconvertible paper standards and fluctuating exchanges are in operation.
  3. (35 points) Explain in detail the effects on Bank A and on the banking system as a whole arising from the deposit in bank A of $100 of newly-printed currency. The deposit is made by a worker who has just received the currency from the government. Assume the bank is fully exploiting its lending power.

_________________

Economics 230
Midquarter Examination
November 5, 1948

  1. (25 points) It has been argued that it would be profitable for a member bank to borrow from its Federal Reserve bank even at a rate of interest considerably higher than the rate the member bank charges to its customers; and that this is so because one dollar of additional reserves can support several dollars of additional deposits. For example, if $1 of additional reserves can support $5 of additional deposits, it is argued that it would be profitable (if we neglect the cost of making the loan) for a member bank that can lend at 6% to borrow at any rate up to 30%. Evaluate this argument.
  2. (25 points)
    1. Nondeposit currency currently in circulation in the United States include Federal Reserve notes, silver certificates, United States notes (greenbacks) and National Bank Notes. In addition, there is a large volume of gold certificates outstanding but not in circulation. Indicate brieflythe historical origin of each of these types of currency, and the major episode in our monetary development each one symbolizes.
    2. What is the FDIC? What, in your view, is its essential function (which may not be the same as its announced purpose) in our current monetary structure?
  3. (50 points) Indicate whether the operation described in the first column would, in the first instance, increase (+), leave unchanged (0), or reduce (-) the item listed at the top of each column. For simplicity, assume (a) that nonmember banks are notinvolved in any of the transactions, (b) that the Treasury deposits all funds received in a Reserve Bank and pays for all expenditures by checks on a Reserve Bank, (c) that all nondeposit currency is in the form of Federal Reserve Notes. Take account only of the essentially bookkeeping effects of the operation, not of subsequent effects. For example, in operation (1) the decline in currency outside banks and the Treasury might so disturb the public’s relative holdings of deposit and nondeposit currency as to lead subsequently to a conversion of deposits into nondeposit currency. Do nottake such subsequent effects into account.
    [+1 for each correct, -1 for each wrong, 0 for no entry]

 

Operation Currency outside banks and Treasury Member bank Federal Reserve Bans
Demand Deposits Excess Reserves Deposits Excess Gold Reserves
Purchase of government bond by public
From Federal Reserve Bank
(1) with non deposit currency [-] [0] [0] [0] [+]
(2) by check [0] [-] [-] [-] [+]
From Treasury
(3) with non deposit currency [-] [0] [0] [+] [0]
(4) by check [0] [-] [-] [0] [0]
From public
(5) with non deposit currency [0] [0] [0] [0] [0]
(6) by check [0] [0] [0] [0] [0]
Purchase of government bond by Treasury from
(7) public a [0]
b [+]
[+]
[0]
[+]
[0]
[0]
[-]
[0]
[0]
(8) member bank [0] [0] [+] [0] [0]
(9) Federal Reserve bank [0] [0] [0] [-] [+]
Conversion of demand deposit by public into
(10) non deposit currency [+] [-] [+] and [0]
[-] and [-]
[only if both]
[0]
(11) time deposit [0] [-] [0] [0] [0]

_________________

Final Examination for Economics 230
Autumn, 1946
3 hours and overnight.

Part I

  1. Define briefly the following terms:
    1. Required reserves
    2. Open market policy
    3. Gold points
    4. Rediscount rate
    5. Inconvertible paper currency
    6. Transactions velocity of circulation
    7. The equation of exchange
  2. What techniques are available to the Federal Reserve System for controlling the total volume of currency? How does each technique work? Under what conditions is each technique likely to be effective?
  3. It is often asserted that in returning to gold at the pre-first-world-war parity Britain “overvalued” the pound. What does this statement mean? What kind of evidence would be required to test its validity and how should this evidence be interpreted? If the statement is true, what effects would overvaluation of the pound be expected to have on Great Britain? What factors would operate to remove these effects and to correct the overvaluation? What kinds of governmental policy, if any, would speed up the process of correcting the overvaluation?

Part II

  1. (20 points) What is the 100% reserve proposal? Discuss its advantages and disadvantages as compared with the present system.
  2. (30 points) A newspaper story of January 21, 1946, on President Truman’s budget message, had the following headlines and first two paragraphs:

“TRUMAN MAPS FIRST DEBT CUT SINCE 1930
CASH ON HAND TO OFFSET ’47 DEFICIT

“Washington—President Truman’s first budget proposes to spend $4,300,000,000 more than the government will collect, but for the first time since 1930, it won’t increase the national debt.
“Mr. Truman proposes to withdraw from the Treasury cash balance sufficient funds not only to offset this deficit but also to reduce the debt by $7,000,000.”

In answering this question assume that Government cash balances are held on deposit in member banks, and that no reserves are required for government balances.

(a) What is the monetary effect of financing the deficit by use of cash balances? Would this effect be deflationary or inflationary compared with such alternatives as raising additional revenue from taxes, or borrowing additional sums from (1) the nonbanking public, (2) member banks, (3) reserve banks.

(b) What is the monetary effect of using cash balances to reduce the debt? Discuss the effects if the bonds are purchased from 81) the nonbanking public, (2) member banks, (3) reserve banks.

_________________

FINAL EXAMINATION, ECONOMICS 230, FALL, 1947

Part I

  1. In speaking of monetary developments in the United States at the beginning of the nineteenth century, H. L. Reed remarks, “the country was so inadequately provided with specie that the advantages of a money economy were not sufficiently extended and diffused.” What do you think this statement means? Does it make sense as it stands? If not, can you suggest an interpretation of it that makes sense?
  2. Explain in detail how, in a fractional reserve system, a given deficit in reserves may force a much larger contraction in currency. In your statement, indicate the factors that set a limit to the contraction and contrast the single bank with the banking system.
  3. To what causes does Gregory attribute the breakdown of the Gold Standard in Great Britain in 1931?

 

Part II

  1.    a. Assume that there is a free market in which English pounds exchange for American dollars. Indicate whether each of the following would, by itself, tend to raise or lower the price of a pound in terms of dollars.

1) An increase in tourist travel by Americans in England. [A. Raise]
2) A rise in dividend payments on American common stocks owned by British. [A. Raise]
3) A sudden craze in Britain for American films leading to increased showings of American films. [A. Lower]
4) Increased repayment by Britain of loans from the U.S. [A. Lower]
5) The raising of abnormally large amounts of relief funds in the United States to finance the shipment of special food packages to Great Britain. [A. Raise]

b. If England and the United States were both on a gold standard what words would it be reasonable to substitute for “raise the price of a pound in terms of dollars”? [A. “ship gold to Britain”] for “lower the price of a pound in terms of dollars”? [A.“ship gold to U.S.”]

c. You are asked what the total amount of money in the United States is. Discuss the problems of definition that would arise, indicating the considerations that would be relevant in deciding what to count as money.

d. Marriner Eccles, chairman of the Federal Reserve Board, recently proposed to Congress that member banks be required to set up a special reserve of 25 per cent of deposit liabilities in addition to existing reserves. Three members of the Federal Advisory Council—a council composed of private bankers who advise the Federal Reserve Board—testified against the proposal. The N. Y. Times reports them as maintaining that “it would reduce loans needed to finance production, and thus prove inflationary.” Discuss this statement.

_________________

Economics 230
Final Examination
December 16, 1949

  1. (100 points) Indicate whether each of the following statements is true (T), false (F), or uncertain (U) and state briefly the reason for your answer.
    1. Legal reserves held by a bank are a liability of the bank.
    2. Banks that are members of the Federal Reserve System may not count cash in their vault as part of their legal reserves.
    3. Every bank that is a member of the Federal Reserve System must carry Federal Deposit Insurance.
    4. Every bank that carries Federal Deposit Insurance must be a member of the Federal Reserve System.

5 and 6. Bank A sells a government bond to Bank B, both banks being members of the Federal Reserve Stem. This…

    1. …increases total member bank reserves.
    2. …does not change total deposit liabilities of member banks.

7, 8, 9. Bank A, which is a member of the Federal Reserve System sells a government bond to Mr. Jones. Bank A deposits the proceeds in its account with a Federal Reserve Bank. This…

    1. …increases total member bank reserves.
    2. …does not change total deposit liabilities of member banks.
    3. …increases the ratio of reserves to deposit liabilities.
      *  *  *  *  *
    4. Since banks can expand loans by several times the amount of excess reserves, a bank that could make additional sound loans at 5 per cent, could afford to pay much more than 5 per cent to induce individuals to deposit currency in the bank, since such a deposit would increase the bank’s excess reserves.
    5. The economic function of legal reserve requirements is to protect depositors in a bank against undue extensions of loans by banks.
    6. An expansion of investments and an expansion of loans by commercial banks have identical effects on the quantity of money.

13 through 16. Mr. Jones pays his Federal income tax with a check on a member bank. The Federal government uses this check to buy a government bond from a Federal Reserve Bank. This operation…

    1. …reduces total member bank deposit liabilities.
    2. …reduces total member bank reserves.
    3. …increases the ratio of member bank reserves to member bank deposit liabilities.
    4. …increases the excess gold reserves of the Federal Reserve System.
      *  *  *  *  *
    5. The post-war rise in prices in the United States was one of the factors making necessary the recent devaluation of the British pound.
    6. Any phenomena that would lead to an outflow of gold from the United States under a gold standard would lead to a fall in the price of the dollar in terms of foreign currencies under a system of inconvertible currencies and flexible exchange rates.

19, 20, 21. Suppose that under an international gold standard, foreign payments and receipts by the United States balance so that there is no net outflow or inflow of gold.

    1. A sudden increase of gifts by residents of the United States to non-residents would tend to lead to an outflow of gold from the United States.
    2. A reduction in the tariffs imposed by France on goods imported into France would tend to lead to an outflow of gold from the United States.
    3. A large technological advance in Great Britain lowering the price of automobiles produced in Great Britain would lead to an outflow of gold from the United States.
      *  *  *  *  *
    4. Under a gold standard, a decline in the rate of interest will tend to narrow the spread between the gold points.
    5. Under existing laws governing bank reserve requirements, a tendency for people to move from farms and small communities to large cities is, by itself, a factor tending to reduce the total quantity of money.
    6. A lengthening in the average pay-period (through, say, an increase in the proportion of workers paid monthly instead of weekly) is, by itself a factor tending to reduce the price level.
    7. K, in the cash balance equation, will be increased by anything that leads people to expect price to fall.
    8. The numerical value of V in the transactions type of equation of exchange depends on the definition of M.
    9. The equation of exchange demonstrates that an increase in the quantity of money must lead to an increase in prices.
    10. Since one man’s receipts are another man’s expenditures, it follows that the quantity of money can be changed only by capital transactions.
    11. The rediscount rate is used by the Federal Reserve system to discourage purchase of government securities.
    12. Monetary policy can be more effective in preventing inflation than in preventing deflation.

 

  1. (50 points) “In the early history of our country there was a dearth of currency and specie. It was difficult to have cash on hand, especially when most of the specie was used to pay for imports.” (E. R. Taus, Central Banking Functions of the United States Treasury, 1789-1941, p. 22).

Discuss the economic meaning of these sentences. Do they make sense as they stand? If so, explain. If not, can you suggest any interpretation of them that does make sense? In your answer, emphasize analysis, not economic history.

  1. (50 points)

“One method proposed for bringing the reserve position under control while protecting the market for government securities held by banks is to require banks to keep a reserve of government securities against deposits, in addition to present cash reserves…..In all essential respects, raising required reserve ratios by adding a security-reserve requirement is identical with a straight increase of cash reserve requirements, combined with an equivalent purchase of government securities by the Reserve Banks. The only significant difference is that the security-reserve proposal provides the member banks with the equivalent of a subsidy (in the form of interest on the bonds) to compensate for the loss of earnings on additional assets tied up as reserves.”
Do you agree? Justify your answer.

  1. (50 points) Under our present monetary system, a desire on the part of the pubic to hold an increased fraction of its money in the form of currency and a decreased fraction in the form of deposits is said to be a factor making for a decrease in the total amount of money (currency plus deposits) in existence. Explain this statement in detail. In your explanation, distinguish between the effect of an outflow of cash on the individual bank and on the system.
  1. (50 points) Currency in public circulation (“cash in pocket”) was approximately one-sixth of the total amount of money (currency plus demand deposits plus time deposits) in the United States in 1892, it fell fairly steadily to about one-twelfth in 1930, and then rose more or less steadily until it is again approximately one-sixth, or about the same level as in 1892. The initial decline was interrupted by a rise during the first World War; and the subsequent rise was accelerated during the second World War.
    How would you explain the long decline to 1930? The subsequent rise? This tendency for the ratio to rise during war time? (Note that there is no unambiguously “right” answer to this question. So far as I know, those movements have not been exhaustively studied, and hypotheses to explain them have not been tested. You are being asked to construct hypotheses about them).

 

Source:  Hoover Institution Archives. Papers of Milton Friedman, Box 76, Folder 8 “University of Chicago, Econ. 230”.

Image Source:  Cropped from a photograph of Milton Friedman, George Stigler, and Aaron Director at the founding meeting of the Mont Pelerin Society, 1947, Milton Friedman papers, Hoover Institution Archives,